Sie sind auf Seite 1von 101

TEST - 1

INDIAN POLITY
SOLUTION PDF

LEARN MAZE
www.learnmaze.com INDIAN POLITY – TEST 1 Learn Maze

Question 1
Which of the following provisions are empowered by the constitution.
1. The Governor can specify the Scheduled Castes in various states.
2. State Legislative Assembly may by law include or exclude the list of various scheduled castes.
Select the correct answer using the codes below.
A. 1 only
B. 2 only
C. Both 1 and 2
D. Neither 1 nor 2
Answer : D
Brief explanation
Statement 1: As per Article 341 President can specify the SC in various states. Governor is just consulted by
the President. So statement 1 is wrong .
Statement 2: Constitution permits this list of notified Scheduled Castes (SC) to be modified by Parliament. So
statement 2 is wrong
Detailed explanation
Article 330 -342 deals with special provisions relating to certain classes
Article 341 deals with Scheduled Castes .
341. Scheduled Castes
(1) The President may with respect to any State or Union territory, and where it is a State after consultation
with the Governor thereof, by public notification, specify the castes, races or tribes or parts of or groups
within castes, races or tribes which shall for the purposes of this Constitution be deemed to be Scheduled
Castes in relation to that State or Union territory, as the case may be
(2) Parliament may by law include in or exclude from the list of Scheduled Castes specified in a notification
issued under clause ( 1 ) any caste, race or tribe or part of or group within any caste, race or tribe, but save as
aforesaid a notification issued under the said clause shall not be varied by any subsequent notification
Recent events related to Article 341
The Constitution (Scheduled Castes) Order (Amendment) Bill, 2017
The Bill amends the Constitution (Scheduled Castes) Order, 1950 and the Constitution (Pondicherry)
Scheduled Castes Order, 1964.

P a g e 1 | 100
www.learnmaze.com INDIAN POLITY – TEST 1 Learn Maze

 Amendments to the 1950 Order: The Bill seeks to modify the list of SCs for the state of Odisha. The
castes Sualgiri, Swalgiri will be introduced as synonyms to the Sabakhia caste in the list of SCs for the
state of Odisha.
 Amendments to the 1964 Order: The name of the Union Territory of Pondicherry was modified to
Puducherry by a central Act in 2006. The Bill seeks to incorporate this change and replace
‘Pondicherry’ with ‘Puducherry’ in the Order.
Source : PRS : http://www.prsindia.org/billtrack/the-constitution-scheduled-castes-orders-amendment-bill-
2017-4667/

Question 2
Which of the following statements regarding Departmentally Related Standing Committees are correct?
1. DRSC’s were formed under the guidance of former PM Nehru in 1961 .
2. They have a have a dedicated set of researchers associated with them.
3. DRSC’s examine bills referred to them; select specific topics related to the ministries and examine
implementation by the Government.
4. Each meeting of DRSC is televised in Lok Sabha TV.
Select the correct answer using the codes given below.
A) 1 and 2 only
B) 2 and 3 only
C) 3 only
D) 1 and 4 only
Answer C
Brief explanation
Statement 1 is incorrect. The DRSCs were formed in 1993; prior to that, there was no systematic process to
examine Bills, and select committees were formed from time to time for some important Bills.
Statement 2 is incorrect. One of the major drawback of this committee is the lack of standing research
support.
Statement 3 is correct
Statement 4 is incorrect. All committees meet behind closed doors and only the final report is published, with
summary minutes.
Detailed explanation
Departmentally Related Standing Committees (DRSCs) perform three important functions:
P a g e 2 | 100
www.learnmaze.com INDIAN POLITY – TEST 1 Learn Maze

1. Examine Bills referred to them;


2. Select specific topics related to the ministries and examine implementation by the Government;
3. Examine the budgetary outlays of the departments.
Their performance affects the overall effectiveness of Parliament as an institution that makes laws, holds the
Government accountable, and gives sanction for public spending.
Objectives of these committees

 They help Parliament manage its business better. It is easier to examine a topic in depth by a
committee of 30 than by an assembly of 700.
 They enable input from experts and those who may be directly affected by a policy or legislation. For
example, the DRSCs often invite comments from the public and call people to testify.
 Being outside direct public glare allows members to discuss issues and reach consensus without
worrying about constituency pressures.
 The anti-defection law does not apply to committees — therefore, decisions are not usually made on
party lines.
 These committees allow members to focus on some specific areas and build their expertise, which
helps them scrutinize issues more thoroughly.
Issues

 The DRSC usually invites experts while scrutinizing Bills. However, this is not always the case, even for
Bills with wide ramifications. For example, the DRSC that examined the Right to Education Bill, 2008 did
not invite any expert witness; this Bill guarantees free education to all children ages six to 14.
 All Bills are not referred to committees. Whereas during the period of the last two parliaments, 60
percent and 71 percent of all Bills were referred to committees, just 27 percent of Bills introduced in
the current Parliament have been so referred.
 The recommendation of committees are not binding. It is for the Government or any other member to
move the relevant amendments, which may then be voted upon by the House. The idea is, committees
are a small part of Parliament which make recommendations, and the full House has the authority and
responsibility to make the final decision.
 One major weakness of these committees is the lack of standing research support. They are backed by
the general support staff of Parliament and do not have a dedicated set of researchers associated with
them.
 A final issue relates to the transparency of the work of committees. All committees meet behind closed
doors and only the final report is published, with summary minutes. There have been arguments that
the meetings should be televised or at least the full transcripts be published. The counter-argument is
committees work as discussion forums and often reach consensus, as there is no pressure on members
to posture for their support base.
Source : http://www.thehindubusinessline.com/opinion/columns/departmentally-related-standing-
committees-india-parliament/article9898334.ece
P a g e 3 | 100
www.learnmaze.com INDIAN POLITY – TEST 1 Learn Maze

DRSC
Main objective – Accountability of executive to Parliament, particularly financial accountability.
Number of committees – 24
Member of this committee should not be minister
Term of office of standing committee – 1 year
Total members – 31 ( 21 from Lok Sabha and 10 from Rajya Sabha )
Source : Indian Polity 5th edition - Page 23.5

Question 3
With reference to Selectorate Theory consider the following statements:
1. It refers to the theory of politics which elaborates on how various kinds of political leaders capture and
hold on to power.
2. It was proposed by American political scientist Kenneth Waltz .
Which of the above statements given above is/are correct?
A. 1 only
B. 2 only
C. Both 1 and 2
D. Neither 1 nor 2
Answer : A
Brief explanation
Statement 1: Selectorate Theory refers to the theory of politics which elaborates on how various kinds of
political leaders capture and hold on to power. So this is a correct statement .
Statement 2: This is incorrect . Selectorate Theory is proposed by American political scientist Bruce Bueno de
Mesquita in a number of works including his 2011 book, The Dictator’s Handbook, co-authored with Alastair
Smith.
Detailed explanation
Selectorate Theory states that leaders primarily care about nothing else but staying in power for as long as
possible, and try to manoeuvre their actions accordingly. Further, it elaborates on five rules that smart leaders
need to follow to extend their stay in power, and what these rules mean for the welfare of society.
Kenneth Waltz - Waltz was a founder of neorealism, or structural realism, in international relations theory.

P a g e 4 | 100
www.learnmaze.com INDIAN POLITY – TEST 1 Learn Maze

Source 1 : http://www.thehindu.com/opinion/op-ed/what-is-selectorate-theory-in-political-
science/article19771579.ece
Source 2 : https://en.wikipedia.org/wiki/Kenneth_Waltz

Question 4
The Charter Act of 1853 provided for which of the following?
A. This act separated the legislative and executive functions of the Governor-General's Council.
B. It ended the system of double government by abolishing the Board of Control and Court of Directors.
C. It ended the activities of East India Company as a commercial body , which became a purely
administrative body .
D. It provided for establishment of new legislative councils for Bengal , North Western Frontier Province
and Punjab .
Answer : A
Brief explanation
Option A is correct .
Option B is incorrect . This is the most important feature of Government of India Act of 1858
Option C is incorrect . It is a feature of Charter Act of 1833
Option D is incorrect . This is the feature of Indian Council Act of 1861 .
Detailed explanation
Features of Charter Act of 1853

 It separated the legislative and executive functions of the Governor-General’s council. It added six new
members called legislative councilors to the council. Thus, it established a separate Governor-General’s
legislative council which came to be known as the Indian (Central) Legislative Council. This legislative
wing of the council functioned as a mini-Parliament, adopting the same procedures as the British
Parliament. Thus, legislation, for the first time, was treated as a special function of the government,
requiring special machinery and special process.
 It introduced an open competition system of selection and recruitment of civil servants. The
covenanted civil service was thus thrown open to the Indians also. Thus, the Macaulay Committee
(Committee on the Indian Civil Service) was appointed in 1854.
 It extended the Company’s rule and allowed it to retain the possession of Indian territories on trust for
the British Crown. But, it did not specify any specific period, unlike the previous Charters. This meant
that the Company’s rule could be terminated at any time the Parliament liked.

P a g e 5 | 100
www.learnmaze.com INDIAN POLITY – TEST 1 Learn Maze

 For the first time, it introduced local representation in the Indian (Central) Legislative Council. 4 out of
6 new members were appointed by the local (provincial) governments of Madras, Bombay, Bengal and
Agra.
Source : Indian Polity – Laxmikanth – 5th edition - Historical Background – P1.4

Question 5
Consider the following statements regarding punitive detention.
1. It is the detention as a punishment for the crime committed by an individual.
2. It is the detention made as a precautionary measure.
Which of the statements given above is/are incorrect?
a) 1 only
b) 2 only
c) Both 1 and 2
d) Neither 1 nor 2
Answer : B
Brief explanation
The question is to identify the incorrect statement . Option A is correct statement regarding Punitive
detention .
Option B is regarding preventive detention and not punitive detention . So this is the incorrect option .
Detailed explanation
Punitive and preventive detention
Crime is an act against the society at large and requires for stricter action from the authorities. The criminal
justice administration allows detention of a person in two circumstances, or a detention can be one of the two
kinds:
Punitive detention, and Preventive detention
Punitive detention is the detention as a punishment for the crime committed by an individual. It takes place
after the actual commission of an offence or at least after an attempt has been made. The time taken from
actual offence to detention can vary in length. It is a punishment imparted to the wrongdoer and involves
strict measures. The duration of such a detention depends on what the law stipulates for the particular
offence.

P a g e 6 | 100
www.learnmaze.com INDIAN POLITY – TEST 1 Learn Maze

Preventive detention is the detention made as a precautionary measure. This kind of detention can be made
by the authorities even on a slight apprehension that the person can commit a crime. It is generally made for
protecting the society from any future happening. It is not a punishment but a precaution. This detention
comes to an end the moment the apprehension of danger ends.
Source : http://sanamurtaza.blogspot.in/2011/08/punitive-and-preventive-detention.html
Criticism of preventive detention – Article 22
http://www.thehindu.com/op/2004/09/07/stories/2004090700101500.htm

Question 6
Which of the following statement is the meaning of the parliamentary term Sine Die ?
A. The act of examining something closely.
B. Involved in or characterized by deliberation and discussion and examination.
C. An act or expression of criticism and censure.
D. With no appointed date for resumption.
Answer : D
Brief explanation
Option A – Scrutiny - The act of examining something closely.
Option B – Deliberative - Involved in or characterized by deliberation and discussion and examination.
Option C – Reprimand - An act or expression of criticism and censure
Option D – Sine Die - With no appointed date for resumption.
Detailed explanation
Definition of sine die
Without any future date being designated (as for resumption) .
Adjournment sine die (from the Latin "without day") means "without assigning a day for a further meeting or
hearing".To adjourn an assembly sine die is to adjourn it for an indefinite period. A legislative body adjourns
sine die when it adjourns without appointing a day on which to appear or assemble again.
Source 1 : https://www.vocabulary.com/lists/358171
Source 2 : https://en.wikipedia.org/wiki/Adjournment_sine_die

Question 7
P a g e 7 | 100
www.learnmaze.com INDIAN POLITY – TEST 1 Learn Maze

Which principle among the following is considered as Gandhian Principle in Directive Principles of State
Policy?
A. To organize agriculture and animal husbandry on modern and scientific lines.
B. To make provision for just and humane conditions for work and maternity relief.
C. To promote voluntary formation, autonomous functioning, democratic control and professional
management of co-operative societies.
D. To take steps to secure the participation of workers in the management of industries.
Answer : C
Brief explanation
Option A : Liberal–Intellectual Principle - Article 48: To organize agriculture and animal husbandry on modern
and scientific lines.
Option B : Socialistic Principle - Article 42: To make provision for just and humane conditions for work and
maternity relief.
Option C : Gandhian Principle - Article 43 B: To promote voluntary formation, autonomous functioning,
democratic control and professional management of co-operative societies.
Option D : Socialistic Principle - Article 43 A: To take steps to secure the participation of workers in the
management of industries.
Detailed explanation
Classification of the Directive Principles
The classification of Directive Principles of State Policy have not been mentioned in the constitution of India.
On the basis of their direction in various perspectives, we can divide them into three categories, i.e. socialistic,
Gandhian and liberal–intellectual.
Socialistic Principles
These principles reflect the ideology of socialism. They lay down the framework of a democratic socialist state,
aim at providing social and economic justice, and set the path towards welfare state.
Following Articles state the guidelines of Socialistic Principles of state policy:
 Article 38: To promote the welfare of the people by securing a social order permeated by justice social,
economic and political and to minimise inequalities in income, status, facilities and opportunities.
 Article 39: To secure
1. the right to adequate means of livelihood for all citizens;

P a g e 8 | 100
www.learnmaze.com INDIAN POLITY – TEST 1 Learn Maze

2. the equitable distribution of material resources of the community for the common good;
3. prevention of concentration of wealth and means of production;
4. equal pay for equal work for men and women;
5. preservation of the health and strength of workers and children against forcible abuse; and
6. opportunities for healthy development of children.
 Article 39 A: To promote equal justice and to provide free legal aid to the poor.
 Article 41: To secure the right to work, to education and to public assistance in cases of
unemployment, old age, sickness and disablement.
 Article 42: To make provision for just and humane conditions for work and maternity relief.
 Article 43: To secure a living wage7, a decent standard of life and social and cultural opportunities for
all workers.
 Article 43 A: To take steps to secure the participation of workers in the management of industries.
 Article 47: To raise the level of nutrition and the standard of living of people and to improve public
health.
Gandhian Principles
These principles are based on Gandhian ideology. They represent the programme of reconstruction
enunciated by Gandhi during the national movement. In order to fulfil the dreams of Gandhi, some of his ideas
were included as Directive Principles.
Following Articles state the guidelines of Gandhian Principles of state policy:
 Article 40: To organise village panchayats and endow them with necessary powers and authority to
enable them to function as units of self-government.
 Article 43: To promote cottage industries on an individual or co-operation basis in rural areas.
 Article 43 B: To promote voluntary formation, autonomous functioning, democratic control and
professional management of co-operative societies.
 Article 46: To promote the educational and economic interests of SCs, STs, and other weaker sections
of the society and to protect them from social injustice and exploitation.
 Article 47: To prohibit the consumption of intoxicating drinks and drugs which are injurious to health.
 Article 48: To prohibit the slaughter of cows, calves and other milch and draught cattle and to improve
their breeds.
Liberal–Intellectual Principles

P a g e 9 | 100
www.learnmaze.com INDIAN POLITY – TEST 1 Learn Maze

The principles counted in this category signify the ideology of liberalism. Following articles state the guidelines
of Liberal–Intellectual Principles of state policy:
 Article 44: To secure for all citizens a uniform civil code throughout the country.
 Article 45: To provide early childhood care and education for all children until they complete the age of
six years.
 Article 48: To organise agriculture and animal husbandry on modern and scientific lines.
 Article 48-A: To protect and improve the environment and to safeguard forests and wild life.
 Article 49: To protect monuments, places and objects of artistic or historic interest which are declared
to be of national importance.
 Article 50: To separate the judiciary from the executive in the public services of the State.
 Article 51: To promote international peace and security and maintain just and honourable relations
between nations; to foster respect for international law and treaty obligations, and to encourage
settlement of international disputes by arbitration.
Source : Indian Polity 5th Edition – Laxmikanth – P 8.2 ,8.3

Question 8
Which among the following are Non – Constitutional Bodies in India
1. National Human Rights Commission
2. Advocate General of the State
3. National Commission for ST’s
4. Central Information Commission
5. NITI Aayog
Select the correct answer using the code given below.
(a) 1, 3 and 4 only
(b) 1, 2 and 4 only
(c) 1, 4 and 5 only
(d) 1, 2, 3 and 5 only
Answer : C
Brief Explanation

P a g e 10 | 100
www.learnmaze.com INDIAN POLITY – TEST 1 Learn Maze

Option 1 - National Human Rights Commission : Non – Constitutional Body


Option 2 - Advocate General of the State : Constitutional Body
Option 3 – National Commission for ST’s : Constitutional Body
Option 4 - Central Information Commission : Non – Constitutional Body
Option 5 - NITI Aayog : Non – Constitutional Body
Detailed Explanation
Constitutional Bodies
1. Election Commission
2. Union Public Service Commission
3. State Public Service Commission
4. Finance Commission
5. National Commission for SC’s
6. National Commission for ST’s
7. Special Officer for Linguistic Minorities
8. Comptroller and Auditor General of India
9. Attorney General of India
10. Advocate General of the State
Non – Constitutional Bodies
1. NITI Aayog
2. National Human Rights Commission
3. State Human Rights Commission
4. Central Information Commission
5. State Information Commission
6. Central Vigilance Commission
7. Central Bureau of Investigation
8. Lokpal and Lokayuktas

Source : Indian Polity 5th Edition – Laxmikanth – P 8.2 ,8.3

Question 9
Which of the following are the recommendations of Rajamannar committee
1. Planning commission should be disbanded and its place should be taken by a statutory body.

P a g e 11 | 100
www.learnmaze.com INDIAN POLITY – TEST 1 Learn Maze

2. All India Services should be abolished.


Which of the above is/are correct?
A 1 only
b) 2 only
c) Both 1 and 2
d) None of the above
Answer : C
Brief explanation
Statement 1 and 2 is correct .
Detailed explanation
Rajamannar Committee
In 1969, the Tamil Nadu Government (DMK) appointed a three-member committee under the chairmanship of
Dr P V Rajamannar to examine the entire question of Centre–state relations and to suggest amendments to
the Constitution so as to secure utmost autonomy to the states.25 The committee submitted its report to the
Tamil Nadu Government in 1971.
The Committee identified the reasons for the prevailing unitary trends (tendencies of centralization) in the
country.
They include:
(i) certain provisions in the Constitution which confer special powers on the Centre;
(ii) one-party rule both at the Centre and in the states;
(iii) inadequacy of states’ fiscal resources and consequent dependence on the Centre for financial assistance;
and
(iv) the institution of Central planning and the role of the Planning Commission.
The important recommendations of the committee are as follows:
(i) An Inter-State Council should be set up immediately;
(ii) Finance Commission should be made a permanent body;
(iii) Planning Commission should be disbanded and its place should be taken by a statutory body;
(iv) Articles 356, 357 and 365 (dealing with President’s Rule) should be totally omitted;
(v) The provision that the state ministry holds office during the pleasure of the governor should be omitted;
P a g e 12 | 100
www.learnmaze.com INDIAN POLITY – TEST 1 Learn Maze

(vi) Certain subjects of the Union List and the Concurrent List should be transferred to the State List; (vii) the
residuary powers should be allocated to the states; and
(viii) All-India services (IAS, IPS and IFS) should be abolished. The Central government completely ignored the
recommendations of the Rajamannar Committee
Source : Indian Polity 5th Edition – Laxmikanth – P 14.12

Question 10
Which of the following were added or changed in the Preamble by the Forty-second Amendment of the
constitution?
1. Secular
2. Unity and Integrity of the Nation
3. Liberty of faith
4. Socialist
Select the correct answer using the code given below.
(a) 1, 3 and 4 only
(b) 1, 2 and 4 only
(c) 1 and 4 only
(d) 4 only
Answer : B
Brief Explanation
42ND amendment added the words "socialist" and "secular" between the words "Sovereign" and
"democratic" and the words "unity of the Nation" were changed to "unity and integrity of the Nation"
Detailed Explanation
The preamble has been amended only once so far. On 18 December 1976, during the Emergency in India, the
Indira Gandhi government pushed through several changes in the Forty-second Amendment of the
constitution.
A committee under the chairmanship of Sardar Swaran Singh recommended that this amendment be enacted
after being constituted to study the question of amending the constitution in the light of past experience.
Through this amendment the words "socialist" and "secular" were added between the words "Sovereign" and
"democratic" and the words "unity of the Nation" were changed to "unity and integrity of the Nation"

P a g e 13 | 100
www.learnmaze.com INDIAN POLITY – TEST 1 Learn Maze

PREAMBLE

“ WE, THE PEOPLE OF INDIA, having solemnly resolved to constitute India into a SOVEREIGN, SOCIALIST,
SECULAR, DEMOCRATIC REPUBLIC and to secure to all its citizens:
JUSTICE, social, economic and political;
LIBERTY of thought, expression, belief, faith and worship;
EQUALITY of status and of opportunity; and to promote among them all
FRATERNITY assuring the dignity of the individual and the unity and integrity of the Nation;
IN OUR CONSTITUENT ASSEMBLY this 26th day of November, 1949, do HEREBY ADOPT, ENACT AND
GIVE TO OURSELVES THIS CONSTITUTION.

Source : http://indiacode.nic.in/coiweb/amend/amend42.htm

Question 11
If Gojri is to be included in the official languages of the Republic of India which of the following schedule of
the Constitution must be amended.
A. Sixth Schedule
B. Seventh Schedule
C. Eighth Schedule
D. Ninth Schedule
Answer : C
Brief Explanation
Option A : Sixth Schedule contains provisions as to the Administration of Tribal Areas in the States of Assam,
Meghalaya, Tripura and Mizoram.
Option B : Seventh Schedule contains the Union list, State list and the concurrent list.
Option C : Eighth Schedule contains the list of recognized languages.
Option D : Ninth Schedule contains provisions as to validation of certain Acts and Regulation.
Recent Issue
Gujjar scholars demand inclusion of Gojri in Eighth Schedule

P a g e 14 | 100
www.learnmaze.com INDIAN POLITY – TEST 1 Learn Maze

Gojri, a tribal language spoken by nearly two million Gujjars, deserves more attention from the government,
scholars from the community said on Sunday, demanding its inclusion in the Eighth Schedule of the
Constitution.
Source : http://www.firstpost.com/india/gujjar-scholars-demand-inclusion-of-gojri-in-eighth-schedule-
research-centre-for-development-of-language-4077449.html
Inclusion of Khasi language
The State Government of Meghalaya decided to constitute an Advisory Committee which will seriously take up
the matter to ensure that the demand for inclusion of the Khasi language into the Eight Schedule of the
Constitution becomes a reality.
Source : https://thenortheasttoday.com/meghalaya-committee-for-inclusion-of-khasi-language-in-8th-
schedule-of-the-constitution-soon/
Detailed Explanation
Indian Constitution Schedules 1 to 12
1. First schedule contains the list of states and union territories and their territories
2. Second schedule contains provisions of the President, Governors of States, Speaker and the Deputy
Speaker of the House of the People and the Chairman and the Deputy Chairman of the Council of
States and the Speaker and the Deputy Speaker of the Legislative Assembly and the Chairman and the
Deputy Chairman of the Legislative Council of a State, the Judges of the Supreme Court and of the High
Courts and the Comptroller and Auditor-General of India the list of states and union territories and
their territories.
3. Third Schedule contains the Forms of Oaths or Affirmations.
4. Fourth Schedule contains provisions as to the allocation of seats in the Council of States.
5. Fifth Schedule contains provisions as to the Administration and Control of Scheduled Areas and
Scheduled Tribes.
6. Sixth Schedule contains provisions as to the Administration of Tribal Areas in the States of Assam,
Meghalaya, Tripura and Mizoram.
7. Seventh Schedule contains the Union list, State list and the concurrent list.
8. Eighth Schedule contains the list of recognized languages.
9. Ninth Schedule contains provisions as to validation of certain Acts and Regulations.
10. Tenth Schedule contains provisions as to disqualification on ground of defection.
11. Eleventh Schedule contains the powers, authority and responsibilities of Panchayats.
12. Twelfth Schedule contains the powers, authority and responsibilities of Municipalities.

Question 12

P a g e 15 | 100
www.learnmaze.com INDIAN POLITY – TEST 1 Learn Maze

In Kesavananda Bharti Vs. State of Kerala ruling Supreme Court says that the basic structure of the
Constitution could not be abrogated even by a constitutional amendment. Which of the below features are
included in the basic structure of Constitution ?
1. Free and fair elections
2. Parliamentary system
3. Rule of Law
4. Judicial Review
Select the correct answer using the codes below.
A. 1 and 2 only
B. 1, 3 and 4 only
C. 2 and 4 only
D. All of the above
Answer : D
Explanation
ELEMENTS OF THE BASIC STRUCTURE
The present position is that the Parliament under Article 368 can amend any part of the Constitution including
the Fundamental Rights but without affecting the ‘basic structure’ of the Constitution. However, the Supreme
Court is yet to define or clarify as to what constitutes the ‘basic structure’ of the Constitution. From the
various judgements, the following have emerged as ‘basic features’ of the Constitution or elements /
components / ingredients of the ‘basic structure’ of the constitution:
1. Supremacy of the Constitution
2. Sovereign, democratic and republican nature of the Indian polity
3. Secular character of the Constitution
4. Separation of powers between the legislature, the executive and the judiciary
5. Federal character of the Constitution
6. Unity and integrity of the nation
7. Welfare state (socio-economic justice)
8. Judicial review
9. Freedom and dignity of the individual

P a g e 16 | 100
www.learnmaze.com INDIAN POLITY – TEST 1 Learn Maze

10. Parliamentary system


11. Rule of law
12. Harmony and balance between Fundamental Rights and Directive Principles
13. Principle of equality
14. Free and fair elections
15. Independence of Judiciary
16. Limited power of Parliament to amend the Constitution
17. Effective access to justice
18. Principles (or essence) underlying fundamental rights.
19. Powers of the Supreme Court under Articles 32, 136, 141 and 142
20. Powers of the High Courts under Articles 226 and 227

Question 13
With reference to Lyngdoh Committee consider the following statements
1. The political parties were completely barred from financing the elections of candidates of their affiliated
student organizations as the students were only permitted to raise small funds from contribution among
themselves.
2. The maximum limit for spending by a student in election is fixed at Rs 10,000.
3. Only those students, who had attained minimum permissible percentage of attendance as prescribed in a
college or university, or 75 per cent where it had not been prescribed, would be eligible to vote.
Which of the statements given above is/are correct?
A. 1 only
B. 1 and 2 only
C. 1 and 3 only
D. All of the above
Answer : C
Explanation
Statement 1 and 3 are correct .
Statement 2 is incorrect . The maximum limit for spending by a student in election is fixed at Rs 5,000.
P a g e 17 | 100
www.learnmaze.com INDIAN POLITY – TEST 1 Learn Maze

Detailed Explanation
The Lyngdoh Committee was appointed by Government of India in 2005 under the chairmanship of the former
Chief Election Commissioner of India, James Michael Lyngdoh, to frame the guidelines on Students’ Union
elections in Colleges/Universities as per the directions issued by the Supreme Court of India. Its report was
submitted by the Committee on May 23, 2006. The Supreme Court in October 2006 ordered that the Lyngdoh
Committee report be implemented in the Students’ Union elections to establish accountability, transparency
and discipline.
Highlights of Recommendations
a) Time Duration: The Committee’s recommendations have clearly mentioned about the time duration for
holding the Union elections. It has recommended that elections be held on a yearly basis and between six-to-
eight weeks from the date of commencement of the academic session.
b) Age: The eligibility criteria have been strictly laid down in order to prioritise educational activities.
Undergraduate students between the ages of 17 and 22 may contest elections. This age range may be
appropriately relaxed in the case of professional Colleges, where courses often range from four to five years.
For Postgraduate Students the maximum age limit to legitimately contest an election would be 24 to 25 years.
For research students the maximum age limit to legitimately contest an election would be 28 years. Marks are
also a criterion because the student’s priority must be given to his/her studies. The candidate should in no
event have any academic arrears in the year of contesting the election. The candidate shall have one
opportunity to contest for the post of an office-bearer, and two opportunities to contest for the post of an
executive member.
c) Attendance: The candidate should have attained the minimum percentage of attendance as prescribed by
the University or 75 per cent attendance, whichever is higher.
d) Previous Criminal Record: The candidate shall not have a previous criminal record, that is to say, he should
not have been tried and/or convicted of any criminal offence or misdemeanour. The candidate shall also not
have been subject to any disciplinary action by the University authorities.
e) Expenditure: The maximum permitted expenditure per candidate shall be Rs 5000.
Source : https://www.mainstreamweekly.net/article1806.html
Recent News : Student Elections in Mumbai , EPW Magazine – September 30 , 2017

Question 14

Consider the following statement in regard with the election of Vice President of India
A. Nominated members of Lok Sabha & Rajya Sabha are not eligible to vote in the Vice-President’s election.

P a g e 18 | 100
www.learnmaze.com INDIAN POLITY – TEST 1 Learn Maze

B. The nomination paper of a candidate has to be signed by at least twenty eligible voters as proposers and
at least twenty eligible voters as seconders.
Which of the above statements is/are correct?
A. 1 only
B. 2 only
C. Both 1 and 2
D. None of the above
Answer : B
Brief Explanation
Statement 1 is incorrect . The nominated members of both the Lok Sabha & the Rajya Sabha are also eligible
to vote in the Vice-President’s election.
The total number of voters in the Vice-Presidential election is 790. The number of voters from the Lok Sabha is
545 (543 elected and 2 nominated) while the number of voters from the Rajya Sabha is 245 (233 elected and
12 nominated).
Statement 2 is correct .
Detailed Explanation
Election of the Vice-President
1. The Vice-President is elected by an electoral college consisting of members of both Houses of Parliament, in
accordance with the system of proportional representation by means of the single transferable vote and the
voting in such election is by secret ballot. The Electoral College to elect a person to the office of the Vice-
President consists of all members of both Houses of Parliament*.
2. The Vice-President is not a member of either House of Parliament or of a House of a Legislature of any
state. If a member of either House of Parliament or of a House of a Legislature of any state is elected as Vice-
President, he is deemed to have vacated his seat in that House on the date he/she enters his office as Vice-
President.
3. A person cannot be elected as Vice-President unless he -
a. is a citizen of India;
b. has completed the age of 35 years, and
c. is qualified for election as a member of the Council of States (Rajya Sabha).
i.e. he should be a citizen of India, 30 years of age and an elector of the Parliamentary constituency in a State
or Union Territory which he seeks to be elected to represent.
P a g e 19 | 100
www.learnmaze.com INDIAN POLITY – TEST 1 Learn Maze

A person is not also eligible if he holds any office of profit under the Government of India or a State
Government or any subordinate local authority.
4. An election to fill a vacancy caused by the expiry of the term of office of Vice-President is completed before
the expiry of the term. In case a vacancy arises by reasons of death, resignation or removal or otherwise, the
election to fill that vacancy is held as soon as possible after the occurrence**. The person so elected is entitled
to hold office for a full term of 5 years from the date he enters office.
Superintendence of the Election of the Vice-President:

The Election Commission of India conducts the election to the office of the Vice-President.
Important Provisions relating to the Election of the Vice-President:
1. The election of the next Vice-President is to be held within 60 days of the expiry of the term of office of the
outgoing Vice-President.
2. The Returning Officer usually appointed to conduct the Vice-Presidential elections is the Secretary-General
of either House of the Parliament, by rotation. The Returning Officer issues a public notice of the intended
election in a prescribed form, inviting nomination of candidates and specifies the place where the nomination
papers are to be delivered.
Any person qualified to be elected and intending to stand for election as Vice-President is required to be
nominated by at least 20 MPs as proposers and at least 20 MPs as seconders.
Nomination papers are to be presented to the Returning Officer at the place and upto the time and date,
specified in the public notice. A maximum of 4 nomination papers by, or on behalf of, any candidate may be
presented to, or accepted by, the Returning Officer.
3. A candidate seeking election as Vice-President is required to make a security deposit of Rs.15,000/-. This is
the only amount that is required to be deposited by a candidate irrespective of the number of nomination
papers filed on his behalf.
4. The nomination papers are scrutinised on the specified date by the Returning Officer in the presence of the
candidate and his proposer or seconder and any one other person duly authorised.
5. Any candidate may withdraw his candidature by a notice in writing in a prescribed form delivered to the
Returning Officer within the time specified.
6. In the election an elector has as many preferences as there are candidates. In casting his vote, an elector is
required to record on his ballot paper the figure 1 at the space opposite the name of the candidate whom he
chooses as his first preference and may, in addition, record as many subsequent preferences as he/she wishes
by recording on his ballot paper the figures 2,3,4, and so on, in the space opposite the names of other
candidates. The votes should be recorded in the international form of Indian numerals or in the Roman form
or in the form of any Indian language but should not be indicated in words.
P a g e 20 | 100
www.learnmaze.com INDIAN POLITY – TEST 1 Learn Maze

Every ballot paper represents one vote at each count. The procedure for counting votes consists of the
following steps:
a. The number of first preference votes secured by each candidate is ascertained.
b. The numbers so ascertained are added up - the total is divided by two and one is added to the quotient
disregarding any remainder. The resulting number is the quota sufficient for a candidate to secure his return
at the election.
c. If at the end of the first or any subsequent count, the total number of votes credited to any candidate is
equal to, or greater than the quota, that candidate is declared elected.
d. If at the end of any count, no candidate can be declared elected, then;
(a) the candidate who upto the stage has been credited with the lowest number of votes shall be excluded
from the poll, and all his ballot papers will be again scrutinised, one by one, with reference to the second
preference marked, if any, on them. These ballot papers will be transferred to the respective remaining
(continuing) candidates for whom such second preferences have been marked thereon, and the value of votes
of those ballot papers credited to such candidates. These ballot papers shall be transferred to the aforesaid
continuing candidate. The ballot papers on which the second preference is not marked shall be treated as
exhausted ballot papers and shall not be counted further, even if they contain third or any subsequent
preference.
If at the end of this count, some candidate reaches the quota, he shall be declared elected.
(b) If at the end of the second count also, no candidate can be declared elected, the counting will proceed still
further by exclusion of the candidate who is now lowest on the poll upto this stage. All his ballot papers,
including the ballot papers which he might have received during the second count, will again be scrutinised
with reference to the 'next available preference' marked on each of them. If on a ballot paper received by him
in the first count, the second preference is marked for any of the continuing candidates, it shall be transferred
to that candidate. If on any such ballot paper, the second preference is marked for the candidate who has
already been excluded in the second round, such ballot paper shall be transferred with reference to the third
preference, if any, for a continuing candidate. Similarly, the ballot papers received by him in the second round
by way of transfer will also be scrutinised with reference to the third preference marked on them.
This process of exclusion of candidates lowest on the poll will be repeated till one of the continuing candidates
reaches the quota.
7. After the election has been held and the votes have been counted, the Returning Officer declares the result
of the election. Thereafter, he reports the result to the Central Government (Ministry of Law & Justice) and
the Election Commission of India and the Central Government publishes the name of the person elected as
Vice-President, in the Official Gazette.
Disputes regarding Election of the Vice-President:

P a g e 21 | 100
www.learnmaze.com INDIAN POLITY – TEST 1 Learn Maze

1. All doubts and disputes arising in connection with the election of the Vice-President are enquired into and
decided by the Supreme Court of India whose decision is final.
2. A petition challenging the election of the Vice-President is heard by a five-judge bench of the Supreme
Court of India.
3. The petition has necessarily to be accompanied by a security deposit of Rs. 20,000/-.
Text of Oath of Affirmation by the Vice-President:
"I, ________ do swear in the name of God that I will bear true faith and allegiance to solemnly affirm the
Constitution of India as by law established and that I will faithfully discharge the duty upon which I am about
to enter."

*The electoral college for electing the President of India comprises of only the elected members of Parliament
and State Legislative Assemblies.
**The Constitution provides an outer limit of six months (Article 62) for election to the office of the President
of India under these circumstances
Source : http://164.100.47.5/Chairman-Rajyasabha/VPElection.htm

Question 15
Which of the following are related to Right to Freedom under the provisions of Articles 19 to 22 of the
Constitution of India?
1. Abolition of Untouchability
2 . Prohibition of traffic in human beings and forced labor
3 . Protection of life and personal liberty
4 . Free and compulsory education to all children of the age of 6-14 years.
Select the correct answer using the codes given below:
A . 1, 2 and 3 only
B . 2, 3 and 4 only
C. 1, 3 and 4 only
D . 3 and 4 only
Answer : D

P a g e 22 | 100
www.learnmaze.com INDIAN POLITY – TEST 1 Learn Maze

Brief Explanation
Statement 1 is incorrect. Abolition of untouchability comes under Article 17 (Right to Equality . )
Statement 2 is incorrect. Article 23 deals with the prohibition of traffic in human beings and forced labor.
Statement 3 is correct. Article 21 deals with Protection of life and personal liberty.
Statement 4 is correct. Article 21A states that that state shall provide free and compulsory education to all
children of the age of 6-14 years.
Detailed Explanation
Right to Freedom (Articles 19-22):
Article 19 (Protection of certain rights regarding freedom of speech, etc.):
Article 19 says that all citizens shall have the right to freedom of speech and expression.
To assemble peacefully and without arms.
To form associations or unions.
To move freely throughout the territory of India.
To practice any profession or to carry on any occupation, trade or business.
Article 20 (Protection in respect of conviction for offenses):
Article 20 says that state can impose reasonable restrictions on the groups of security of the state, friendly
relations with foreign states, public order, decency, morality, contempt of court, defamation, etc.
Article 21 deals with Protection of life and personal liberty.
Article 21A states that that state shall provide free and compulsory education to all children of the age of 6-14
years.
Article 22 deals with protection against arrest and detention in certain cases.
Source : https://edugeneral.org/blog/polity/fundamental-rights-articles-14-18-19-22-23-24-25-28-29-30-
32/#article_19-22

Question 16
In which of the following circumstances the President uses his individual discretion in appointing Prime
Minister
1 . When no political party has clear majority in Lok Sabha.
2 . When the Prime Minister in office dies suddenly and there is no obvious successor

P a g e 23 | 100
www.learnmaze.com INDIAN POLITY – TEST 1 Learn Maze

Which of the above is/are correct?


A. 1 only
B. 2 only
C. Both 1 and 2
D. None of the above
Answer : C
Brief Explanation
Statement 1 and 2 are correct.
Detailed Explanation
Discretionary powers of the President
The President of India almost always acts on the aid and advice of the Council of Ministers except under the
following circumstances where he acts on his discretion:

 In appointing the Prime Minister from among the contenders when no single party attains majority
after elections to the Lok Sabha.
 A Council of Ministers is voted out and after resigning advises the President to dissolve, the Lok Sabha
and hold fresh elections (or resigns and advises so without being voted out). The President is expected
to exercise his discretion in such circumstances as much of the Lok Sabha's life may still be intact and it
is worthwhile to explore the possibility of forming an alternative Government.
 While exercising a pocket veto.
 Disqualifying members of the legislature when the Council's advice is not taken.
 Can return the advice of the Council of Ministers once for its reconsideration.
 Can return the Bill passed by the Parliament once for its reconsideration.
Source : Indian Polity 5th edition – Laxmikanth

Question 17
The 61st Amendment of Constitution of India is significant because
A . Voting age is reduced from 21 to 18.
B . Introduction of Panchayati Raj, addition of Part IX to the Constitution.
C . Right to Property deleted from the list of fundamental rights.
D . Defection to another party after election made illegal.
P a g e 24 | 100
www.learnmaze.com INDIAN POLITY – TEST 1 Learn Maze

Answer : A
Brief Explanation
Option A is correct . The Sixty-first Amendment' of the Constitution of India, officially known as The
Constitution (Sixty-first Amendment) Act, 1989, lowered the voting age of elections to the Lok Sabha and to
the Legislative Assemblies of States from 21 years to 18 years. This was done by amending Article 326 of the
Constitution, which concerns to elections to the Lok Sabha and the Assemblies.
Option B, C,D are incorrect .
Detailed Explanation
Important Amendments to Indian Constitution.

Amendment Year Importance

Reorganisation of states on linguistic basis and


7 1956 abolition of Class A, B, C and D states and
introduction of Union Territories.

Adjustments to Indian territory as a result of


9 1960
agreement with Paksitan.

Dadra, Nagar and Haveli included in Indian


10 1961 Union as a Union Territory on acquisition from
Portugal.

Goa, Daman and Diu included in Indian Union


12 1961 as a Union Territory on acquisition from
Portugal.

The state of Nagaland formed with special


13 1963
protection under Article 371A.

Pondicherry incorporated into Indian Union


14 1962
after transfer by France.

21 1967 Sindhi added as language in the 8th schedule.

Privy purse paid to former rulers of princely


26 1971
states abolished.

P a g e 25 | 100
www.learnmaze.com INDIAN POLITY – TEST 1 Learn Maze

Amendment Year Importance

36 1975 Sikim included as an Indian state.

Fundamental Duties prescribed, India became


42 1976
Socialist Secular Republic.

Right to Property deleted from the list of


44 1978
fundamental rights.

Defection to another party after election


52 1985
made illegal.

61 1989 Voting age reduced from 21 to 18.

Konkani, Manipuri and Nepali added as


71 1992
languages in the Eighth Schedule.

Introduction of Panchayati Raj, addition of


73 1993
Part IX to the Constitution.

Introduction of Nagarpalikas and


74 1993
Municipalities.

Free and compulsory education to children


86 2002
between 6 and 14 years.

Bodo, Dogri, Santhali and Maithli added to the


92 2003 list of recognised languages. Service Tax
introduced.

1960, 1970, Extension of reservation of seats for SC/ST and


8,23,45,62, 79
1980, 1989, nomination of Anglo-Indian members in
and 95
2000 and 2010 Parliament and State Assemblies.

Substituted Odia for Oriya in the Eighth


96 2011
Schedule to the Constitution

Introduction of Part IXB in the Constitution


97 2012
pertaining to Co-operative Societies

101 2016 Introduction of Goods and Services Tax (GST)

P a g e 26 | 100
www.learnmaze.com INDIAN POLITY – TEST 1 Learn Maze

Question 18
Which of the following acts are considered as contempt of Houses of Parliament .
1 . Questioning the character or the impartiality of the Speaker / Chairman in the discharge of his duties.
2 . Publication of false or distorted reports of the proceedings .
3 . Alleging partiality in the report of a Parliamentary Committee .
Select the correct answer using the codes given below:
A . 1 and 2 only
B . 1 and 3 only
C . 2 and 3 only
D . All of the above
Answer : D
Brief Explanation
All the 3 Statements are correct
Detailed Explanation
Contempt of the House may be defined generally as "any act or omission which obstructs or impedes either
House of Parliament in the performance of its functions, or which obstructs or impedes any Member or officer
of such House in the discharge of his duty, or which has a tendency directly or indirectly, to produce such
results." It may be stated that it is not possible to enumerate exhaustively every act which might be construed
by the House as a contempt of the House. Some of the important types of contempt of Parliament are,
however, mentioned below:-
(i) Speeches or writings reflecting on the House, its Committees or Members;
(ii) Reflections on the character and impartiality of the Chairman/Speaker in the discharge of his duty;
(iii) Publication of false or distorted report of the Proceedings of the House;
(iv) Publication oil expunged proceedings of the House;
(v) Publication of proceedings of secret Sessions of the House;
(vi) Pre-mature publication of proceedings, evidence or report of a Parliamentary Committee;
(vii) Reflections on the report of a Parliamentary Committee;

P a g e 27 | 100
www.learnmaze.com INDIAN POLITY – TEST 1 Learn Maze

(viii) Molestation of Members on account of their conduct in the House or obstructing Members while in
the performance of their duties as Members or while on their way to or coming after, attending the House or
a Committee thereof;
(ix) Offering bribes to Members to influence them in their Parliamentary conduct;
(x) Intimidation of Members in connection with their Parliamentary conduct;
(xi) Any misconduct or undignified behaviour on the part of a Member, such as, corruption in the
execution of his office as Member, disorderly and undignified conduct contrary to the usage or inconsistent
with accepted standards of Parliamentary conduct;
(xii) Obstructing or molesting officers of the House in the discharge of their duties;
(xiii) Giving false or misleading evidence or information deliberately to the House or a Committee thereof,
by a Member or a witness.
(xiv) Obstructing or molesting any witness during his evidence before a Committee of the House.

Source : Our Parliament – Subash Kashyap

Question 19
Consider the following statements regarding Privilege motion:
1 . The "Privilege Motion" can be moved against a minister only
2 . The Speaker/Chairman has authority to admit or dismiss the Privilege Motion.
Which of the above is/are correct?
A. 1 only
B. 2 only
C. Both 1 and 2
D. None of the above
Answer : B
Brief Explanation
Statement 1 is incorrect . The "Privilege Motion" can be moved against an individual, any in-house committee
or even the entire house.
Statement 2 is correct
P a g e 28 | 100
www.learnmaze.com INDIAN POLITY – TEST 1 Learn Maze

Detailed explanation
The privilege motion is moved by a member when he feels that a minister or any member has committed a
breach of privilege of the House or one or more of its members by withholding facts of a case or by giving
wrong or distorted facts. It can be also termed as a notice by any member of either House of a state legislature
or Parliament, against anyone who is accused of breach of privilege.
A breach of privilege is an infringement of any of the privileges of MPs or Parliament. Parliamentary privilege
refers to the rights and immunities enjoyed by Parliament as an institution and Member of Parliament in their
individual capacity, without which they cannot discharge their functions as endowed upon them by the
Constitution.
Privilege motion can be moved by any lawmaker, or MP against anyone accused of breaching privileges. The
purpose privilege motion is to censure the concerned minister or any other member. The Lok Sabha and the
Rajya Sabha, has separate Privilege Committees.

Question 20
Which of the following veto is exercised by the President of United States of America and not by Indian
President .
A . Qualified Veto
B . Absolute Veto
C . Suspensive Veto
D . Pocket Veto
Answer : A
Explanation
Of the above four, the President of India is vested with three—absolute veto, suspensive veto and pocket
veto. There is no qualified veto in the case of Indian President; it is possessed by the American President.
The veto power enjoyed by the executive in modem states can be classified into the following four types:
1. Absolute veto, that is, withholding of assent to the bill passed by the legislature.
2. Qualified veto, which can be overridden by the legislature with a higher majority.
3. Suspensive veto, which can be over ridden by the legislature with an ordinary majority.
4. Pocket veto, that is, taking no action on the bill passed by the legislature.

P a g e 29 | 100
www.learnmaze.com INDIAN POLITY – TEST 1 Learn Maze

Question 21
Consider the following statements regarding Right to Information Act, 2005:
1 . A citizen can ask any information from any PIO whether the PIO is possessor of that information or not.

2 . PIO should provide information to the appellant in administrative language.


Which of the above is/are correct?
A. 1 only
B. 2 only
C. Both 1 and 2
D. None of the above

Answer : C
Explanation
Statement 1 is correct .
Any PIO Clause: Under this clause a citizen can ask any information from any PIO whether the PIO is possessor
of that information or not. In the disposal of such cases lots of time and energy is wasted due to
administrative hierarchy. Situations becomes much more difficult when 30 Days Clause is enforced with Any
PIO Clause.
Statement 2 is correct .
Language Clause: Under this clause the PIO should provide information to the appellant in administrative
language. It should be noted that administrative language has its own terminology, abbreviations and
complexity which is not possible for a citizen to understand.

Question 22
Who among the following is the chairman of the Inter-state Council in India?
A . Union Home Minister
B . Cabinet Secretary
C . Finance Minister

P a g e 30 | 100
www.learnmaze.com INDIAN POLITY – TEST 1 Learn Maze

D . Prime Minister
Answer : D
Explanation
Prime Minister is the CHAIRMAN of Inter State Council .
The Inter State Council is an Indian constitutional body set up on the basis of provisions in Article 263 of the
Constitution of India. The body was formed by a Presidential Order dated 28 May 1990 on recommendation of
Sarkaria Commission. The Council is formed to discussing or investigating policies, subjects of common
interest, and disputes, among states.
Recent News : According to a gazette notification, the Union ministers who will be members of the
reconstituted council are Rajnath Singh (Home), Sushma Swaraj (External Affairs), Arun Jaitley (Finance), Nitin
Gadkari (Road Transport), Thaawar Chand Gehlot (Social Justice and Empowerment) and Nirmala Sitharaman
(Defence).
Source : https://timesofindia.indiatimes.com/india/inter-state-council-reconstituted-with-pm-modi-as-
chairman/articleshow/61395536.cms

Question 23
Article 35A of Indian Constitution provides which of the below provisions
1. Autonomous status to the state of Jammu and Kashmir.
2. Empowers the Jammu and Kashmir state's legislature to define “permanent residents” of the state and
provide special rights and privileges to those permanent residents.
Select the correct answer from the above statements
A. 1 only
B. 2 only
C. Both 1 and 2
D. None of the above
Answer : B
Explanation
Statement 1 is incorrect .Article 370 of the Indian constitution is an article that gives autonomous status to the
state of Jammu and Kashmir.

P a g e 31 | 100
www.learnmaze.com INDIAN POLITY – TEST 1 Learn Maze

Statement 2 is correct. Article 35A is a provision incorporated in the Constitution giving the Jammu and
Kashmir Legislature a carte blanche to decide who all are ‘permanent residents’ of the State and confer on
them special rights and privileges in public sector jobs, acquisition of property in the State, scholarships and
other public aid and welfare. The provision mandates that no act of the legislature coming under it can be
challenged for violating the Constitution or any other law of the land.

Recent News
http://www.greaterkashmir.com/news/kashmir/supreme-court-defers-hearing-on-article-35-a/264619.html

Question 24
With reference to National Commission to Review the Working of the Constitution consider the following
recommendations.
1 . Sikhism should be treated as a separate religion
2 . Establishment of Inter – Faith commission
3 . MP Local Area Development Scheme should be promoted.
Which of the statements given above is/are not correct?
A . 1 and 2 only
B . 2 and 3 only
C . 1 and 3 only
D 3 only
Answer : D
Explanation
Statement 1 , 2 are correct so the options A ,B and C are incorrect .
Statement 3 is incorrect. The important recommendation of National Commission to Review the Working of
the Constitution is to discontinue MPLAD .

Question 25
A political party is recognized as a national party only if:

P a g e 32 | 100
www.learnmaze.com INDIAN POLITY – TEST 1 Learn Maze

1 . It is recognized as a state party in five states


2 . It wins two per cent of seats in the Lok Sabha at a general election; and these candidates are elected from
three states.
Which of the above is/are correct?
A. 1 only
B. 2 only
C. Both 1 and 2
D. None of the above
Answer : B
Brief Explanation
Statement 1 is incorrect . A political party is recognized as a national party only if it is recognized as a state
party in four states.
Statement 2 is correct .
Detailed Explanation
Conditions for Recognition as a National Party
At present (2016), a party is recognised as a national party if any of the following conditions is fulfilled
1. If it secures six per cent of valid votes polled in any four or more states at a general election to the Lok
Sabha or to the legislative assembly; and, in addition, it wins four seats in the Lok Sabha from any state or
states; or
2. If it wins two per cent of seats in the Lok Sabha at a general election; and these candidates are elected
from three states; or
3. If it is recognized as a state party in four states.

Question 26
Consider the following statements about Central Bureau of Investigation ( CBI ) :
1 . CBI is a statutory body
2 . It derives powers from Delhi Special Police Establishment Act , 1946
3 . It is established on the recommendations of the Santhanam Committee on Prevention of Corruption.
P a g e 33 | 100
www.learnmaze.com INDIAN POLITY – TEST 1 Learn Maze

Which of the following statements given above is/are correct?


A . 1 only
B . 1 and 3 only
C . 2 and 3 only
D . 3 only
Answer : C
Brief Explanation
Statement 1 is incorrect . The CBI is not a statutory body. It derives its powers from the Delhi Special Police
Establishment Act, 1946.
Statement 2 and 3 is correct .
Detailed Explanation
ESTABLISHMENT OF CBI
The Central Bureau of Investigation (CBI) was set up in 1963 by a resolution of the Ministry of Home Affairs.
Later, it was transferred to the Ministry of Personnel and now it enjoys the status of an attached office1. The
Special Police Establishment (which looked into vigilance cases) setup in 1941 was also merged with the CBI.
The establishment of the CBI was recommended by the Santhanam Committee on Prevention of Corruption
(1962-1964). The CBI is not a statutory body. It derives its powers from the Delhi Special Police Establishment
Act, 1946.
The CBI is the main investigating agency of the Central Government. It plays an important role in the
prevention of corruption and maintaining integrity in administration. It also provides assistance to the Central
Vigilance Commission and Lokpal.

Question 27
With reference to Human Immunodeficiency Virus and Acquired Immune Deficiency Syndrome (Prevention
and Control) Act, 2017 consider the following statements:
1 . Every HIV infected or affected person below the age of 18 years has the right to reside in a shared
household and enjoy the facilities of the household.
2 . The requirement for HIV testing as a pre-requisite for obtaining employment is allowed .
Which of the above statements is/are correct?
A. 1 only
B. 2 only
P a g e 34 | 100
www.learnmaze.com INDIAN POLITY – TEST 1 Learn Maze

C. Both 1 and 2
D. None of the above
Answer : A
Brief Explanation
Statement 1 is correct. Every HIV infected or affected person below the age of 18 years has the right to reside
in a shared household and enjoy the facilities of the household. The Bill also prohibits any individual from
publishing information or advocating feelings of hatred against HIV positive persons and those living with
them.
Statement 2 is incorrect . The requirement for HIV testing as a pre-requisite for obtaining employment or
accessing health care or education is prohibited.
Detailed Explanation
The Human Immunodeficiency Virus And Acquired Immune Deficiency Syndrome (Prevention And Control)
Bill, 2014

 The Human Immunodeficiency Virus and Acquired Immune Deficiency Syndrome (Prevention and
Control) Bill, 2014 was introduced in the Rajya Sabha on February 11, 2014 by the Minister for Health
and Family Welfare, Mr. Ghulam Nabi Azad. The Bill seeks to prevent and control the spread of HIV
and AIDS, prohibits discrimination against persons with HIV and AIDS, provides for informed consent
and confidentiality with regard to their treatment, places obligations on establishments to safeguard
their rights, and creates mechanisms for redressing their complaints.
 Prohibition of discrimination against HIV positive persons: The Bill lists the various grounds on which
discrimination against HIV positive persons and those living with them is prohibited. These include the
denial, termination, discontinuation or unfair treatment with regard to: (i) employment, (ii)
educational establishments, (iii) health care services, (iv) residing or renting property, (v) standing for
public or private office, and (vi) provision of insurance (unless based on actuarial studies). The
requirement for HIV testing as a pre-requisite for obtaining employment or accessing health care or
education is also prohibited.
 Every HIV infected or affected person below the age of 18 years has the right to reside in a shared
household and enjoy the facilities of the household. The Bill also prohibits any individual from
publishing information or advocating feelings of hatred against HIV positive persons and those living
with them.
 Informed consent and disclosure of HIV status: The Bill requires that no HIV test, medical treatment,
or research will be conducted on a person without his informed consent. No person shall be
compelled to disclose his HIV status except with his informed consent, and if required by a court
order.

P a g e 35 | 100
www.learnmaze.com INDIAN POLITY – TEST 1 Learn Maze

 Informed consent for an HIV test will not be required in case of screening by any licensed blood bank,
a court order, medical research, and epidemiological purposes where the HIV test is anonymous and
not meant to determine the HIV status of a person. Establishments keeping records of information of
HIV positive persons shall adopt data protection measures.
 Role of the central and state governments: The central and state governments shall take measures
to: (i) prevent the spread of HIV or AIDS, (ii) provide anti-retroviral therapy and infection management
for persons with HIV or AIDS, (iii) facilitate their access to welfare schemes especially for women and
children, (iv) formulate HIV or AIDS education communication programmes that are age appropriate,
gender sensitive, and non stigmatizing, and (v) lay guidelines for the care and treatment of children
with HIV or AIDS. Every person in the care and custody of the state shall have right to HIV prevention,
testing, treatment and counselling services.
 Role of the Ombudsman: An ombudsman shall be appointed by each state government to inquire into
complaints related to the violation of the Act and the provision of health care services. The
Ombudsman shall submit a report to the state government every six months stating the number and
nature of complaints received, the actions taken and orders passed.
 Guardianship: A person between the age of 12 to 18 years who has sufficient maturity in
understanding and managing the affairs of his HIV or AIDS affected family shall be competent to act as
a guardian of another sibling below 18 years of age. The guardianship will be apply in matters relating
to admission to educational establishments, operating bank accounts, managing property, care and
treatment, amongst others.
 Court proceedings: Cases relating to HIV positive persons shall be disposed off by the court on a
priority basis. In any legal proceeding, if an HIV infected or affected person is a party, the court may
pass orders that the proceedings be conducted (a) by suppressing the identity of the person, (b) in
camera, and (c) to restrain any person from publishing information that discloses the identity of the
applicant. When passing any order with regard to a maintenance application filed by an HIV infected
or affected person, the court shall take into account the medical expenses incurred by the applicant.

Source : http://www.prsindia.org/billtrack/the-humanimmunodeficiency-virus-and-acquired-immune-
deficiency-syndrome-prevention-and-control-bill-2014-3126/

Question 28
Which of the following pairs about Indian Constitution is/are correctly matched
Constitutional Provision Borrowed From
1 . Directive Principles of State Policy - Ireland
2 . Quasi-federal system - Canada
3 . Concept of “procedure established by Law - United States
P a g e 36 | 100
www.learnmaze.com INDIAN POLITY – TEST 1 Learn Maze

4 . Fundamental Rights are suspended during Emergency - South Africa


Select the correct answer using the codes given below
A . 1,2 and 3 only
B . 2 , 3 and 4 only
C . 1 . 3 and 4 only
D . 1 and 2 only
Answer : D
Brief Explanation
Option 1 and 2 is correct.
Option 3 and 4 are incorrectly matched . Concept of “procedure established by Law is borrowed from
Japanese constitution . Fundamental Rights are suspended during Emergency - This provision is borrowed
from German constitution
Detailed Explanation
Provision borrowed from various countries
Britain
1. Parliamentary government
2. Rule of Law
3. Legislative procedure
4. Single citizenship
5. Cabinet system
6. Prerogative writs
7. Parliamentary privileges
8. Bicameralism
Ireland
1. Directive Principles of State Policy
2. Nomination of members to Rajya Sabha
3. Method of election of president
Unites States of America
1.Impeachment of the president
2.Functions of president and vice-president
3.Removal of Supreme Court and High court judges
4.Fundamental Rights
P a g e 37 | 100
www.learnmaze.com INDIAN POLITY – TEST 1 Learn Maze

5.Judicial review
6.Independence of judiciary
7.Preamble of the constitution
The National Anthem of India: Crucial Facts at a Glance
Canada
1. Federation with a strong Centre
2. Vesting of residuary powers in the Centre
3. Appointment of state governors by the Centre
4. Advisory jurisdiction of the Supreme Court
Australia
1. Concurrent List
2. Freedom of trade
3. Commerce and inter-course
4. Joint sitting of the two Houses of Parliament
Soviet Constitution (USSR, now Russia)
1. Fundamental duties
2. The ideal of justice (social, economic and po-i Russia) litical) in the Preamble
France
1. The ideals of Republic in the Preamble
2. The ideals of liberty in the Preamble
3. The ideals of equality in the Preamble
4. The ideals of fraternity in the Preamble
Weimar Constitution of Germany
1. Suspension of Fundamental Rights during Emergency
South African Constitution
1. Procedure for amendment of the Constitution
2. Election of members of Rajya Sabha
Japan
1.Concept of “procedure established by Law”
Sources
NCERT – Class XI – Political Science

P a g e 38 | 100
www.learnmaze.com INDIAN POLITY – TEST 1 Learn Maze

Indian Polity – Laxmikanth

Question 29
Which of the following changes are needed to ensure free and fair elections all over India
1 . Removal of Voter Verifiable Paper Audit Trail System.
2 . Photos of candidates in EVMs and Ballot Papers.
Which of the above is/are correct?
A. 1 only
B. 2 only
C. Both 1 and 2
D. None of the above
Answer: B
Brief Explanation

Voter Verifiable Paper Audit Trail System is very good to ensure free and fair elections So Statement 1 is
incorrect.
Statement 2 is correct. Photos of candidates in EVMs is good to ensure candidates with similar names do not
make advantage out of it .
Detailed explanation
What are VVPAT machines?
The Voter Verified Paper Audit Trail is a method that provides feedback to voters. It is an independent
verification printer machine and is attached to electronic voting machines. It allows voters to verify if their
vote has gone to the intended candidate.
How do VVPAT machines work?
When a voter presses a button in the EVM, a paper slip is printed through the VVPAT. The slip contains the
poll symbol and name of the candidate. It allows the voter to verify his/her choice. After being visible to the
voter from a glass case in the VVPAT for seven seconds, the ballot slip will be cut and dropped into the drop
box in the VVPAT machine and a beep will be heard. VVPAT machines can be accessed by polling officers only.
Source : http://www.thehindu.com/news/national/all-you-need-to-know-about-vvpat/article18077550.ece

P a g e 39 | 100
www.learnmaze.com INDIAN POLITY – TEST 1 Learn Maze

Question 30
Which of the following are the recommendations of L.M. Singhvi Committee
1 . Nyaya panchayats should be established for a cluster of villages .
2 . Panchayati Raj Institutions must be constitutionally recognized .
3 . A state finance commission should be set up in states . It should provide guidelines for devolution of
finances to PRI .
Select the correct answer using the codes given below:
A . 1 and 2 only
B . 1 and 3 only
C . 2 and 3 only
D . All of the above
Answer : A
Explanation
Statement 1 and 2 are correct . It is the important recommendation of L.M. Singhvi Committee .
Statement 3 is incorrect . It is the recommendation of Thungon Committee .
L.M. Singhvi Committee .
Its recommendations were -

1. Constitutional recognition to the Panchayati Raj institutions.

2. For it, a new chapter should be added in the Constitution of India.

3. Thus their identity and integrity will be reasonably and substantially inviolate.

4. Constitutional provisions to ensure regular, free and fair elections to the Panchayati Raj bodies.

5. Nyaya Panchayats should be created for a cluster of villages.

6. Reorganize villages to make Gram Panchayats more viable.

7. Emphasized importance of the Gram Sabha; called it as the embodiment of direct democracy.
P a g e 40 | 100
www.learnmaze.com INDIAN POLITY – TEST 1 Learn Maze

8. More financial resources for Village Panchayats

9. Judicial tribunals in each state to adjudicate controversies about election to the Panchayati Raj institutions,
their dissolution and other matters related to their functioning.

Question 31
Which among the following are the features of the Parliamentary form of Government in India?
1 . Conflict between legislature and executive
2 . Stable government
3 . Government run by subject experts
4 . Definiteness in policies
Select the correct answer using the codes given below:
A . 1, 2, 3 and 4
B . 1, 2 and 4
C . 1, 3 and 4
D . None of the above
Answer : D
Explanation
All the above are features of Presidential Form of Government .

P a g e 41 | 100
www.learnmaze.com INDIAN POLITY – TEST 1 Learn Maze

Question 32
Consider the following statements regarding Chairman of State Legislative Council
1 . The salary and allowance of the Chairman are charged on the Consolidated Fund of the State.
2 . The Chairman decides whether a bill is money bill or not and his decision on this question is final .
3 . Chairman is elected by the council itself from amongst its members .
Select the correct answer using the codes given below:
A . 1 and 2 only
B . 2 and 3 only
C . 1 and 3 only
D . 3 only
Answer : C
Expalnation
Statement 1 is correct . The salary and allowance of the Chairman are charged on the Consolidated Fund of
the State and thus are not subject to the annual vote of the state legislature .
Statement 2 is incorrect . The Speaker of State Legislative Assembly decides whether a bill is money bill or not
and his decision on this question is final .

P a g e 42 | 100
www.learnmaze.com INDIAN POLITY – TEST 1 Learn Maze

Statement 3 is correct . Chairman is elected by the council itself from amongst its members . State Legislative
Assembly has no role in it .

Question 33
Which of the following provisions can be amended by simple majority of the parliament ?
1 . Amendment of seventh schedule of constitution .
2 . Admission or establishment of new states .
3 . Delimitation of constituencies .
4 . Use of English language in parliament .
5 . Citizenship - acquisition and termination .
Select the correct answer using the codes given below:
A . 1 , 2 , 3 and 4 only
B . 2 ,3 , 4 and 5 only
C . 1 ,3 , 4 and 5 only
D . All of the above
Answer : B
Explanation
Option 1 – 7th schedule can be amended only by special majority of Parliament and consent of states .
Option 2 ,3 ,4 and 5 can be amended by simple majority
A number of provisions in the Constitution can be amended by a simple majority of the two houses of
Parliament outside the scope of Article 368. These provisions include:
 Admission or establishment of new states.
 Formation of new states and alteration of areas, boundaries or names of existing states.
 Abolition or creation of legislative councils in states.
 Second Schedule-emoluments,
 Allowances, privileges and so on of the president, the governors, the Speakers, judges, etc.
 Quorum in Parliament.

P a g e 43 | 100
www.learnmaze.com INDIAN POLITY – TEST 1 Learn Maze

 Salaries and allowances of the members of Parliament.


 Rules of procedure in Parliament.
 Privileges of the Parliament, its members and its committees.
 Use of English language in Parliament.
 Number of puisne judges in the Supreme Court.
 Conferment of more jurisdiction on the Supreme Court.
 Conferment of more jurisdiction on the Supreme Court.
 Citizenship-acquisition and termination.
 Elections to Parliament and state legislatures.
 Delimitation of constituencies.
 Union territories
 Fifth Schedule-administration of scheduled areas and scheduled tribes.
 Sixth Schedule-administration of tribal areas.

Question 34
Consider the following statements regarding imposition of martial law .
1 . It suspends the government and ordinary law courts
2 . It can be imposed only on three grounds – war , external aggression or armed rebellion .
Which of the above is/are correct ?
A. 1 only
B. 2 only
C. Both 1 and 2
D. None of the above
Answer : A
Explanation
Statement 1 is correct . Martial Law suspends the government and ordinary law courts

P a g e 44 | 100
www.learnmaze.com INDIAN POLITY – TEST 1 Learn Maze

Statement 2 is incorrect . National Emergency can be imposed only on three grounds – war , external
aggression or armed rebellion . Martial Law can be imposed to restore the breakdown of law or order due to
any reason.

Question 35
In regard with Citizenship, consider the following statements:
1 . Naturalized citizen is eligible for the office of President in United States
2 . India deals with various provisions of citizenship in Part III of Indian Constitution .
3 . Friendly aliens enjoy protection against arrest and detention in India
Select the correct answer using the codes given below:
A . 1 and 2 only
B . 1 and 3 only
C . 2 and 3 only
D . 3 only
Answer : D
Explanation
Statement 1 is incorrect . Naturalized citizen is eligible for the office of President in India but not in United
States .
Statement 2 is incorrect . India deals with various provisions of citizenship in Part II of Indian Constitution (
Article 5 – 11 ) .
Statement 3 is correct . Friendly aliens enjoy protection against arrest and detention in India . Enemy aliens
does not enjoy this protection .

Question 36
With reference to Mental Healthcare Act, 2017 consider the following statements ?
1 . The Bill ensures every person shall have a right to access mental health care and treatment from mental
health services run or funded by the appropriate government.
2 . The Bill empowers the government to set-up Central Mental Health Authority at national-level and State
Mental Health Authority in every State.
Which of the above is/are correct?
A. 1 only
P a g e 45 | 100
www.learnmaze.com INDIAN POLITY – TEST 1 Learn Maze

B. 2 only
C. Both 1 and 2
D. None of the above
Answer : C
Explanation
Statement 1 and 2 is correct .
Mental Healthcare Act, 2017
The Mental Healthcare Bill, 2016, aims to provide provide for mental healthcare and services for persons with
mental illness and ensure these persons have the right to live a life with dignity by not being discriminated
against or harassed.

Question 37
Which of the following committee is elected only from members of Lok Sabha ?
A . Estimates Committee
B . Committee on Public Undertakings
C . Public Accounts Committee
D . Joint Committee on Offices of Profit
Answer : A
Explanation
The Estimates Committee, constituted for the first time in 1950, is a Parliamentary Committee consisting of
30 Members, elected every year by the Lok Sabha from amongst its Members. The Chairman of the
Committee is appointed by the Speaker from amongst its members. A Minister cannot be elected as a
member of the Committee and if a member after his election to the Committee, is appointed a Minister,
he ceases to be a member of the Committee from the date of such appointment.

Question 38
Which of the below statement aptly defines suzerainty ?
A . It is the full right and power of a governing body over itself, without any interference from outside sources
or bodies.
B . It is the political, economic, or military predominance or control of one state over others.

P a g e 46 | 100
www.learnmaze.com INDIAN POLITY – TEST 1 Learn Maze

C . It is a system of government that is centralized and dictatorial and requires complete subservience to the
state.
D . It is a situation in which a powerful region or people controls the foreign policy and international relations
of a tributary vassal state while allowing the subservient nation internal autonomy.
Answer : D
Explanation
Option A is incorrect – It is the definition of Sovereignty
Option B is incorrect –It explains about Hegemony
Option C is incorrect – It is regarding Totalitarianism
Option D is correct . The terminology gradually became generalised to refer to any relationship in which one
region or people controls the foreign policy and international relations of a tributary state, while allowing the
tributary nation to have internal autonomy. Suzerainty differs from true sovereignty, as the tributary
state/person is technically independent, and enjoys self-rule (though usually limited in practice).

Question 39
Mahadayi Inter – State Water Dispute Tribunal is concerning which of the following states
1 . Maharashtra
2 . Goa
3 . Karnataka
4 . Telengana
5 . Gujarat
6 . Madhya Pradesh
Select the correct answer using the codes given below:
A . 1 , 2 and 3 only
B . 3 and 4 only
C . 1 , 5 and 6 only
D . 1 and 3 only
Answer : A
Explanation

P a g e 47 | 100
www.learnmaze.com INDIAN POLITY – TEST 1 Learn Maze

Mahadayi or Mhadei river, is described as the lifeline of the Indian state of Goa. The sharing of the waters of
this river is a cause of dispute between the governments of Karnataka, Maharashtra and Goa.

Question 40
Which of the following are unitary features of Indian Constitution ?
1 . All India Services
2 . Division of Power between centre and states
3 . Appointment of Governor
Select the correct answer using the codes given below:
A . 1 and 2 only
B . 1 and 3 only
C . 2 and 3 only
D . All of the above
Answer : B
Brief Explanation
Option 1 and 3 are unitary features .
Option 2 is federal feature of Indian Constitution .
Detailed Explanation
Unitary Features of Indian Constitution
1. Strong Centre
The division of powers is in favour of the Centre and highly inequitable from the federal angle. Firstly, the
Union List contains more subjects than the State List. Secondly, the more important subjects have been
included in the Union List. Thirdly, the Centre has overriding authority over the Concurrent List. Finally, the
residuary powers have also been left with the Centre, while in the USA, they are vested in the states. Thus, the
Constitution has made the Centre very strong.
2. States Not Indestructible
Unlike in other federations, the states in India have no right to territorial integrity. The Parliament can by
unilateral action change the area, boundaries or name of any state. Moreover, it requires only a simple
majority and not a special majority. Hence, the Indian Federation is “an indestructible Union of destructible
states”. The American Federation, on the other hand, is described as “an indestructible Union of indestructible
states”.
P a g e 48 | 100
www.learnmaze.com INDIAN POLITY – TEST 1 Learn Maze

3. Single Constitution:-
Usually, in a federation, the states have the right to frame their own Constitution separate from that of the
Centre. In India, on the contrary, no such power is given to the states. The Constitution of India embodies not
only the Constitution of the Centre but also those of the states. Both the Centre and the states must operate
within this single-frame. The only exception in this regard is the case of Jammu and Kashmir which has its own
(state) Constitution.
4. Flexibility of the Constitution:-
The process of constitutional amendment is less rigid than what is found in other federations. The bulk of the
Constitution can be amended by the unilateral action of the Parliament, either by simple majority or by special
majority. Further, the power to initiate an amendment to the Constitution lies only with the Centre. In US, the
states can also propose an amendment to the Constitution.
5. No Equality of State Representation:-
The states are given representation in the Rajya Sabha on the basis of population. Hence, the membership
varies from 1 to 31. In US, on the other hand, the principle of equality of representation of states in the Upper
House is fully recognised. Thus, the American Senate has 100 members, two from each state. This principle is
regarded as a safeguard for smaller states.
6. Emergency Provisions:-
The Constitution stipulates three types of emergencies—national, state and financial. During an emergency,
the Central government becomes all powerful and the states go into the total control of the Centre. It
converts the federal structure into a unitary one without a formal amendment of the Constitution. This kind of
transformation is not found in any other federation.
7. Single Citizenship:-
In spite of a dual polity, the Constitution of India, like that of Canada, adopted the system of single citizenship.
There is only Indian Citizenship and no separate state citizenship. All citizens irrespective of the state in which
they are born or reside enjoy the same rights all over the country. The other federal states like US, Switzerland
and Australia have dual citizenship, that is, national citizenship as well as state citizenship.
8. Integrated Judiciary:-
The Indian Constitution has established an integrated judicial system with the Supreme Court at the top and
the state high courts below it. This single system of courts enforces both the Central laws as well as the state
laws. In US, on the other hand, there is a double system of courts whereby the federal laws are enforced by
the federal judiciary and the state laws by the state judiciary.
9. All-India Services:-
In US, the Federal government and the state governments have their separate public services. In India also,
the Centre and the states have their separate public services. But, in addition, there are all-India services (IAS,
P a g e 49 | 100
www.learnmaze.com INDIAN POLITY – TEST 1 Learn Maze

IPS, and IFS) which are common to both the Centre and the states. The members of these services are
recruited and trained by the Centre which also possess ultimate control over them. Thus, these services
violate the principle of federalism under the Constitution.
10. Integrated Audit Machinery:-
The Comptroller and Auditor-General of India audit the accounts of not only the Central government but also
those of the states. But, his appointment and removal is done by the president without consulting the states.
Hence, this office restricts the financial autonomy of the states. The American Comptroller-General, on the
contrary, has no role with respect to the accounts of the states.
11. Parliament’s Authority Over State List:-
Even in the limited sphere of authority allotted to them, the states do not have exclusive control. The
Parliament is empowered to legislate on any subject of the State List if Rajya Sabha passes a resolution to that
effect in the national interest. This means that the legislative competence of the Parliament can be extended
without amending the Constitution. Notably, this can be done when there is no emergency of any kind.
12. Appointment of Governor:-
The governor, who is the head of the state, is appointed by the President. He holds office during the pleasure
of the President. He also acts as an agent of the Centre. Through him, the Centre exercises control over the
states. The American Constitution, on the contrary, provided for an elected head in the states. In this respect,
India adopted the Canadian system.
13. Integrated Election Machinery:-
The Election Commission conducts elections not only to the Central legislature but also to the state
legislatures. But, this body is constituted by the President and the states have no say in this matter. The
position is same with regard to the removal of its members as well. On the other hand, US has separate
machineries for the conduct of elections at the federal and state levels.
14. Veto Over State Bills:-
The governor is empowered to reserve certain types of bills passed by the state legislature for the
consideration of the President. The President can withhold his assent to such bills not only in the first instance
but also in the second instance. Thus, the President enjoys absolute veto (and not suspensive veto) over state
bills. But in US and Australia, the states are autonomous within their fields and there is no provision for any
such reservation.

Question 41
Swaran Singh Committee recommended the incorporation of which of the following provisions regarding
Fundamental Duties .
P a g e 50 | 100
www.learnmaze.com INDIAN POLITY – TEST 1 Learn Maze

1 . Duty to pay taxes should also be fundamental duty of citizens .


2 . The Parliament may provide for the imposition of penalty for non – compliance to observe duties .
Which of the above is/are correct?
A. 1 only
B. 2 only
C. Both 1 and 2
D. None of the above
Answer : C
Explanation
Both statements are correct . But these recommendations were not accepted by the Congress Party .
Detailed Explanation
Fundamental Duties.-
It shall be the duty of every citizens of India-
(a) to abide by the Constitution and respect its ideals and institutions, the National Flag and the National
Anthem;
(b) to cherish and follow the noble ideals which inspired our national struggle for freedom;
(c) to uphold and protect the sovereignty, unity and integrity of India;
(d) to defend the country and render national service when called upon to do so;
(e) to promote harmony and the spirit of common brotherhood amongst all the people of India transcending
religious, linguistic and regional or sectional diversities; to renounce practices derogatory to the dignity of
women;
(f) to value and preserve the rich heritage of our composite culture;
(g) to protect and improve the natural environment including forests, lakes, rivers and wild life, and to have
compassion for living creatures;
(h) to develop the scientific temper, humanism and the spirit of inquiry and reform;
(i) to safeguard public property and to abjure violence;
(j) to strive towards excellence in all spheres of individual and collective activity so that the nation constantly
rises to higher levels of endeavour and achievement.]

P a g e 51 | 100
www.learnmaze.com INDIAN POLITY – TEST 1 Learn Maze

Question 42
Which of the following Directive Principle were added as part of 42nd amendment of Constitution of India in
1976 ?
A . To promote equal justice and to provide free legal aid to the poor .
B . To organize agriculture and animal husbandry on modern and scientific lines .
C . To raise the level of nutrition and the standard of living of people and to improve public health
D . To make provisions for just and humane conditions of work and maternity relief.
Answer : A
Explanation
Option A – Article 39 A - Added as 42nd amendment to constitution. To promote equal justice and to provide
free legal aid to the poor .
Option B - Article 48 - To organize agriculture and animal husbandry on modern and scientific lines .
Option C - Article 47 - To raise the level of nutrition and the standard of living of people and to improve public
health
Option D – Article 42 - To make provisions for just and humane conditions of work and maternity relief.

Question 43
Consider the following statement about the office of the President of India.
1 . The office of President becomes vacant only if he is impeached or if his election is declared void .
2 . The nominated members of Lok Sabha and Rajya Sabha cannot participate in his impeachment process .
Which of the above statement is/are correct?
A. 1 only
B. 2 only
C. Both 1 and 2
D. None of the above
Answer : D

P a g e 52 | 100
www.learnmaze.com INDIAN POLITY – TEST 1 Learn Maze

Statement 1 is incorrect . Vacancy in the office of President can occur in any of the following ways .
- On his expiry of tenure of 5 years
- By his resignation
- On his removal by the process of impeachment
- By his death
- Disqualified to hold office when his election is declared void .
Statement 2 is incorrect . The nominated members of Lok Sabha and Rajya Sabha does not participate in the
election of President but can participate in his impeachment process .

Question 44
Which of the following Bills are passed after Joint Sitting of two Houses
1 . Dowry Prohibition Bill ,1960
2 . Banking Service Commission ( Repeal ) Bill , 1977
3 . Prevention of Terrorism Bill , 2002
Select the correct answer using the codes given below:
A . 1 and 2 only
B . 1 and 3 only
C . 2 and 3 only
D . All of the above
Answer : D
Explanation
All the 3 bills mentioned above are passed after joint sitting of Parliament . Joint Sitting of Parliament has been
invoked only thrice .

Question 45
Consider the following statements regarding Railway Budget of India
1 . To railway budget was separated from general budget to introduce flexibility in railway finance

P a g e 53 | 100
www.learnmaze.com INDIAN POLITY – TEST 1 Learn Maze

2 . Railway Finances was separated in 1921 on the recommendations of Acworth Committee .


Which of the above is/are correct?
A. 1 only
B. 2 only
C. Both 1 and 2
D. None of the above
Answer : C
Explanation
Both statements are correct . It was presented as separate budget till 2016 . NDA government decided
to merge both the budgets .
Railway budget was separated from general budget to

Question 46
Which of the following statements is/are correct regarding implementation of National Emergency?
1 . A national emergency powers the Parliament to make laws on the 66 subjects of the State List .
2 . Fundamental Rights can be suspended under Article 360 Emergency provisions .
3 . The proclamation of National Emergency must be approved by both the Houses of Parliament within one
month from the date of its issue .
Select the correct answer using the codes given below:
A . 1 and 2 only
B . 1 and 3 only
C . 2 and 3 only
D . All of the above
P a g e 54 | 100
www.learnmaze.com INDIAN POLITY – TEST 1 Learn Maze

Answer : B
Explanation
Statement 1 is correct. A national emergency powers the Parliament to make laws on the 66 subjects of the
State List . So there can be a lot of power abuse which affects the federal structure during emergency .
Statement 2 is incorrect. Article 360 deals with Financial emergency . Fundamental Rights cannot be
suspended under Article 360 .
Statement 3 is correct. Earlier the approval period was two months . But it is reduced to one month after the
44th amendment in 1978 .

Question 47
Constitutional position of the governor differs from that of President in which of the following aspects.
1 . The Constitution envisages that the possibility of the governor acting at times in his discretion, no such
possibility has been envisaged for the President.
2 . Ministerial advice has been made binding on the President, but no such provision has been made with
respect to the governor.
Which of the above is/are correct?
A. 1 only
B. 2 only
C. Both 1 and 2
D. None of the above
Answer : C
Explanation
Both Statement 1 and 2 is correct .
Constitutional position of the governor differs from that of the president in the following two respects:
1. While the Constitution envisages the possibility of the governor acting at times in his discretion, no
such possibility has been envisaged for the President.
2. After the 42nd Constitutional Amendment (1976), ministerial advice has been made binding on the
President, but no such provision has been made with respect to the governor.

P a g e 55 | 100
www.learnmaze.com INDIAN POLITY – TEST 1 Learn Maze

The Constitution makes it clear that if any question arises whether a matter falls within the governor’s
discretion or not, the decision of the governor is final and the validity of anything done by him cannot be
called in question on the ground that he ought or ought not to have acted in his discretion.

Question 48
Which among the following does not come under the expenditure 'charged' upon the Consolidated Fund of
India?
A . Salaries and allowance of Judges of Supreme Court , High Court and District Courts .
B . Salaries and allowances of Speaker and the Deputy Speaker of the Lok Sabha.
C . The debt charges for which the Government of India is liable, including interest, sinking fund charges and
redemption charges and other expenditure relating to the raising of loans and the service and redemption of
debt.
D . Salaries, allowances and pension of the chairman and members of the Union Public Service Commission.
Answer : A
Explanation
Option A is incorrect .
Expenditure 'charged' upon the Consolidated Fund of India includes only

 Pensions of the judges of high courts.


 Salaries, allowances and pensions of the judges of the Supreme Court.
Option B,C and D are 'charged' upon the Consolidated Fund of India .

Question 49
Which among the following commission was appointed to identify the creamy layer among the OBC’s ?
A . Kalelkar Commission
B . B P Mandal Commission
C . Ram Nandan Committee
D . Balwant Rai Mehta Committee
Answer : C

P a g e 56 | 100
www.learnmaze.com INDIAN POLITY – TEST 1 Learn Maze

Explanation
Option A is incorrect . Adhering to Article 340 of the Constitution of India, the First Backward Classes
Commission was set up by a presidential order on 29 January 1953 under the chairmanship of Kaka Kalelkar.
Option B is incorrect. The Mandal Commission, or the Second Backward Classes Commission, was established
in India on 1 January 1979 by the Janata Party government under Prime Minister Morarji Desai with a
mandate to "identify the socially or educationally backward. In 1980, the Commission's report upheld the
affirmative action practice under Indian law by recommending that members of Other Backward Classes (OBC)
be granted reservations to 27 per cent of jobs under the Central government and public sector undertakings.
Option C is correct . Ram Nandan committee was appointed to identify the creamy layer among the OBC’s . It
submitted its report in 1993 .
Option D is incorrect . Balwant Rai Mehta Committee provided recommendations for decentralization of
governance .

Question 50
Which of the following is/are correct regarding Shadow Cabinet ?
1 . Shadow Cabinet is a feature of the Westminster system of government.
2 . Shadow Cabinet's responsibility is to formulate the policies for the government.
Which of the above is/are correct?
A. 1 only
B. 2 only
C. Both 1 and 2
D. None of the above
Answer : A
Explanation
Statement 1 is correct . Shadow Cabinet is a feature of the Westminster system of government.
Statement 2 is incorrect . Shadow Cabinet's responsibility is to criticise the policies and actions of the
government, as well to offer an alternative program.
Shadow cabinet’ is an unique institution of the British cabinet system. It is formed by the opposition party to
balance the ruling cabinet and to prepare its members for future ministerial office. There is no such institution
in India.
P a g e 57 | 100
www.learnmaze.com INDIAN POLITY – TEST 1 Learn Maze

Question 51
Which of the following writ can be issued by Supreme Court if a public official has refused to perform an
official duty ?
A . Prohibition
B . Certiorari
C . Quo Warranto
D . Mandamus
Answer : D
Explanation

Question 52
Which of the following is/are true about First Past the Post System?

P a g e 58 | 100
www.learnmaze.com INDIAN POLITY – TEST 1 Learn Maze

1 . In India the Presidential Election is based on First Past the Post system
2 . In this system a candidate who secure at least 50 % of the total votes is declared as the winner .
3 . It is the easiest election method used in countries with large population .
Select the correct answer using the codes given below:
A . 1 and 2 only
B . 1 and 3 only
C . 2 only
D . 3 only
Answer : D
Explanation
Statement 1 is incorrect. In India the Presidential Election is based on Proportional Representation System.
Statement 2 is incorrect. In First Past the Post System the candidate who secure highest votes is the winner . If
the highest vote among 20 candidates is 12.5 % , then that person is the winner . There is no need of 50 %
votes to win the election.
Statement 3 is correct . Proportional Representation is a very difficult process . First Past the Post System is
very easy in countries like India . But the minorities are the biggest losers in FPTP system .

Question 53
Consider the following statements regarding a money bill
1 . Authority to decide whether a bill is money bill or not lies with the Speaker of Lok Sabha .
2 . Recently introduced The Appropriation (No. 4) Bill, 2017 is a money bill.
3 . The Money Bill can be introduced only by a ruling party Lok Sabha MP .
Select the correct answer using the codes given below:
A . 1 and 2 only
B . 1 and 3 only
C . 2 and 3 only
D . All of the above

P a g e 59 | 100
www.learnmaze.com INDIAN POLITY – TEST 1 Learn Maze

Answer : A
Explanation
Statement 1 is correct . If any question arises whether a bill is a money bill or not, the decision of the Speaker
of the Lok Sabha is final. His decision in this regard cannot be questioned in any court of law or in the either
House of Parliament or even the president.
Statement 2 is correct . Under Article 114(3), no amount can be withdrawn from the Consolidated Fund of
India without the enactment of such a law by Parliament.
After the Demands for Grants are voted by the Lok Sabha, Parliament's approval for the withdrawal from the
Consolidated Fund of India of the amounts so voted and of the amount required to meet the expenditure
charged on the Consolidated Fund is sought through the Appropriation Bill.
Statement 3 is incorrect . The Money Bill can be introduced only by a Minister .

Question 54
Which among the following is enacted in pursuance of Directive Principle of State Policy?
1 . Equal Remuneration Act 1976
2 . Legal Services Authority Act 1987
3 . Wildlife Protection Act 1972
4 . Factories Act 1948
Select the correct answer using the codes given below.
A . 1, 2 and 3 only
B . 1 and 3 only
C . 1, 2 and 4 only
D . All of the above
Answer : D
Explanation
All the above Acts were enacted in pursuance of DPSP .
Please refer 8.7 -8.8 Indian Polity 5th edition Laxmikanth for complete List .

P a g e 60 | 100
www.learnmaze.com INDIAN POLITY – TEST 1 Learn Maze

Question 55
Consider the following statements about Finance Commission of India .
1 . Established by the President of India under Article 112 of the Indian Constitution.
2 . 14th Finance Commission recommends the share of states in the net proceeds of the shareable Central
taxes should be 42%.
Which of the above is/are correct?
A. 1 only
B. 2 only
C. Both 1 and 2
D. None of the above
Answer : B
Brief Explanation
Statement 1 is incorrect . The Finance Commission was established by the President of India in 1951 under
Article 280 of the Indian Constitution.
Statement 2 is correct.
Detailed Explanation
Major Recommendations of 14th Finance Commission headed by Prof. Y V Reddy
1. The share of states in the net proceeds of the shareable Central taxes should be 42%. This is 10
percentage points higher than the recommendation of 13th Finance Commission.
2. Revenue deficit to be progressively reduced and eliminated.
3. Fiscal deficit to be reduced to 3% of the GDP by 2017–18.
4. A target of 62% of GDP for the combined debt of centre and states.
5. The Medium Term Fiscal Plan(MTFP) should be reformed and made the statement of commitment
rather than a statement of intent.
6. FRBM Act need to be amended to mention the nature of shocks which shall require targets relaxation.
7. Both centre and states should conclude 'Grand Bargain' to implement the model Goods and Services
Act(GST).

P a g e 61 | 100
www.learnmaze.com INDIAN POLITY – TEST 1 Learn Maze

8. Initiatives to reduce the number of Central Sponsored Schemes(CSS) and to restore the predominance
of formula based plan grants.
9. States need to address the problem of losses in the power sector in time bound manner.

Question 56
‘Instrument of Accession of Jammu and Kashmir to India’ was signed by Pandit Jawaharlal Nehru and
Maharaja Hari Singh on 26 October 1947. Under this, the state surrendered which of the following subjects
to the Dominion of India.
1 . Defence
2 . External Affairs
3 . Communications
Select the correct answer using the codes given below.
A . 1 and 2 only
B . 1 and 3 only
C . 2 and 3 only
D . All of the above
Answer : D
Explanation
All the three matters are transferred to Dominion of India .

The ‘Instrument of Accession of Jammu and Kashmir to India’ was signed by Pandit Jawaharlal Nehru and
Maharaja Hari Singh on 26 October 1947. Under this, the state surrendered only three subjects (defence,
external affairs and communications) to the Dominion of India. At that time, the Government of India made a
commitment that ‘the people of this state, through their own Constituent Assembly, would determine the
internal Constitution of this state and the nature and extent of the jurisdiction of the Union of India over the
state, and until the decision of the Constituent Assembly of the State, the Constitution of India could only
provide an interim arrangement regarding the state. In pursuance of this commitment, Article 370 was
incorporated in the Constitution of India. It clearly states that the provisions with respect to the State of J&K
are only temporary and not permanent.

P a g e 62 | 100
www.learnmaze.com INDIAN POLITY – TEST 1 Learn Maze

Question 57
As per the table of precedence arrange the officials from higher to lower rank
1 . Chief Justice of India
2 . Deputy Prime Minister
3 . Vice Chairperson , NITI Aayog
4 . Former President
Select the correct answer using the codes given below.
A. (2),(4),(3),(1)
B. (2),(4),(1),(3)
C. (4),(2),(1),(3)
D. (1),(2),(4),(3)
Answer : C
Explanation
Image of Top 8 ranks provided.

P a g e 63 | 100
www.learnmaze.com INDIAN POLITY – TEST 1 Learn Maze

Question 58
Which of the following amendment added Sindhi as the 15 th language in the Eighth Schedule of the
constitution ?
A . Seventeenth Amendment Act , 1964
B . Twenty First Amendment Act , 1967
C . Thirty Sixth Amendment Act , 1975
D . Forty Ninth Amendment Act , 1984
Answer : B
Explanation
Option A . Seventeenth Amendment Act , 1964 –
1. Prohibited the acquisition of land under personal cultivation unless the market value of the land is paid as
compensation.
2. Included 44 more Acts in the Ninth Schedule.
Option B . Twenty First Amendment Act , 1967 - Added Sindhi as the 15th language in the Eighth Schedule of
the constitution.
Option C . Thirty Sixth Amendment Act , 1975 - Made Sikkim a full-fledged State of the Indian Union and
omitted the Tenth Schedule.
Option D . Forty Ninth Amendment Act , 1984 - Gave a constitutional sanctity to the Autonomous
District Council in Tripura.

Question 59
Consider the following statements regarding Panchayati Raj system in India:
1 . 73rd Amendment Act provides constitutional status to Panchayati Raj Institutions .
2 . Electoral Rolls of all elections to the Panchayat is prepared by Election Commission of India .
Which of the above is/are correct?
A. 1 only
B. 2 only
P a g e 64 | 100
www.learnmaze.com INDIAN POLITY – TEST 1 Learn Maze

C. Both 1 and 2
D. None of the above
Answer : A
Explanation
Statement 1 is correct . The Amendment Act has added part IX to the Constitution of India entitled as
‘Panchayats’. The part consists of provisions from Article 243 to 243-0. A new schedule called as Eleventh
Schedule lists 29 functional items that panchayats are supposed to deal with under Article 243-G.
Statement 2 is incorrect . The superintendence, direction and control of the preparation of electoral rolls and
the conduct of all elections to the municipalities shall be vested in the state election commission.

Question 60
Consider the following statements regarding Leader of Opposition in Lok Sabha .
1. In order to get formal recognition, the concerned party must have at least 10% of the total strength of the
House
2 . It is not compulsory to have a Formal Leader of Opposition in Lok Sabha .
Which of the above is/are correct?
A. 1 only
B. 2 only
C. Both 1 and 2
D. None of the above
Answer : C
Explanation
Statement 1 is correct .The Leader of the Opposition is the politician who leads the official opposition in either
House of the Parliament of India. in order to get formal recognition, the concerned party must have at least
10% of the total strength of the House (55 seats in the Lok Sabha).
Statement 2 is correct . If any party fails to get 10% seats in opposition, the House will not have recognised
leader of the opposition. A single party has to meet the 10% seat criteria, not an alliance.

P a g e 65 | 100
www.learnmaze.com INDIAN POLITY – TEST 1 Learn Maze

Question 61
Under Article 28 religious instruction is permitted in which of the following type of educational institutions.
A . Institutions wholly maintained by the State.
B . Institutions administered by the State but established under any endowment or trust.
C . Institutions recognized by the State.
D . Institutions receiving aid from the State.
Answer : B
Explanation
In (a) religious instruction is completely prohibited while in (b), religious instruction is permitted. In (c) and (d),
religious instruction is permitted on a voluntary basis.

Question 62
Part XVII of Indian Constitution deals with which of the following provisions.
A . Emergency Provisions
B . Amendment of the Constitution
C . Special provisions relating to certain classes
D . Official Language
Answer : D
Explanation

Part Subject Articles


Part I The Union and its territory Art. 1 to 4
Part II Citizenship Art. 5 to 11
Part III Fundamental Rights Art. 12 to 35
Part IV Directive Principles Art. 36 to 51
Part IVA Fundamental Duties Art. 51A
Part V The Union Art. 52 to 151
Part VI The States Art. 152 to 237
P a g e 66 | 100
www.learnmaze.com INDIAN POLITY – TEST 1 Learn Maze

Part Subject Articles


Part VII Repealed by Const. (7th Amendment) Act, 1956
Part VIII The Union Territories Art. 239 to 242
Part IX The Panchayats Art. 243 to 243O
Art. 243P to
Part IXA The Muncipalities
243ZG
Art. 243ZH to
Part IXB The Co-operative Societies
243ZT
Part X The Scheduled and Tribal Areas Art. 244 to 244A
Part XI Relations between the Union and the States Art. 245 to 263
Part XII Finance, Property, Contracts and Suits Art. 264 to 300A
Trade, Commerce and Intercourse within the Territory
Part XIII Art. 301 to 307
of India
Part XIV Services under the Union and the States Art. 308 to 323
Part
Tribunals Art. 323A to 323B
XIVA
Part XV Elections Art. 324 to 329A
Part XVI Special provisions relating to certain classes Art. 330 to 342
Part
Official Language Art. 343 to 351
XVII
Part
Emergency Provisions Art. 352 to 360
XVIII
Part XIX Miscellaneous Art. 361 to 367
Part XX Amendment of the Constitution Art. 368
Part XXI Temporary, Transitional and Special Provisions Art. 369 to 392
Part Short title, commencement, authoritative text in Hindi
Art. 393 to 395
XXII and repeals

Question 63
What is the difference between “judicial review” and “judicial activism”
1 .It is a legal maxim that suggests that no one is above the law and governmental decisions must be made
only by applying known legal and moral principles .
2 . Judicial activism denotes the proactive role played by the judiciary in the protection of the rights of citizens
and in the promotion of justice in the society.
P a g e 67 | 100
www.learnmaze.com INDIAN POLITY – TEST 1 Learn Maze

Which of the above is/are correct?


A. 1 only
B. 2 only
C. Both 1 and 2
D. None of the above
Answer : B
Brief Explanation
Statement 1 is incorrect . It is the explanation of Rule of Law
Statement 2 is correct .
Detailed Explanation
Judicial activism is defined in the following way:
1. “Judicial activism is a way of exercising judicial power that motivates judges to depart from normally
practiced strict adherence to judicial precedent in favour of progressive and new social policies. It is commonly
marked by decision calling for social engineering, and occasionally these decisions represent intrusion in the
legislative and executive matters”.
2. “Judicial activism is the practice in the judiciary of protecting or expanding individual rights through
decisions that depart from established precedent, or are independent of, or in opposition to supposed
constitutional or legislation intent”.

Question 64
The President can remove the Central Vigilance Commissioner from the office by himself
1 . If he has been convicted of an offence which involves a moral turpitude.
2 . If he is concerned or interested in any contract made by the Central government.
3 . If he is adjudged an insolvent
4 . If he engages , during his term of office , in any paid employment outside the duties of his office .
Select the correct answer using the codes given below:
A . 1, 2 and 3 only
B . 1, 3 and 4 only

P a g e 68 | 100
www.learnmaze.com INDIAN POLITY – TEST 1 Learn Maze

C . 2, 3 and 4 only
D . 1, 2, 3 and 4
Answer : B
Brief Explanation
Statement 1 , 3 and 4 is correct .
Statement 2 is incorrect . In this case the president can also remove the Central Vigilance Commissioner or
any vigilance commissioner only on the ground of proved misbehaviour or incapacity.
Detailed Explanation
The president can remove the Central Vigilance Commissioner or any vigilance commissioner from the office
under the following circumstances:
(a) If he is adjudged an insolvent; or
(b) If he has been convicted of an offence which (in the opinion of theCentral government) involves a moral
turpitude; or
(c) If he engages, during his term of office, in any paid employment outside the duties of his office; or
(d) If he is (in the opinion of the president), unfit to continue in office by reason of infirmity of mind or body;
or
(e) If he has acquired such financial or other interest as is likely to affect prejudicially his official functions.
In addition to these, the president can also remove the Central Vigilance Commissioner or any vigilance
commissioner on the ground of proved misbehaviour or incapacity. However, in these cases, the president has
to refer the matter to the Supreme Court for an enquiry. If the Supreme Court, after the enquiry, upholds the
cause of removal and advises so, then the president can remove him. He is deemed to be guilty of
misbehaviour, if he
(a) is concerned or interested in any contract or agreement made by the Central government, or (b)
participates in any way in the profit of such contract or agreement or in any benefit or emolument arising
therefrom otherwise than as a member and in common with the other members of an incorporated company.

Question 65
Consider the following statements.
Assertion (A) : The National Human Rights Commission is a statutory body .

P a g e 69 | 100
www.learnmaze.com INDIAN POLITY – TEST 1 Learn Maze

Reason (R) : It is established in 1993 under a legislation enacted by Parliament namely Protection of Human
Rights Act 1993 .
In the context of the above, which of these is correct?
A . A is correct, and R is an correct explanation of A.
B . A is correct, but R is not a correct explanation of A.
C . A is correct, but R is incorrect.
D . Both A and R are incorrect.
Answer : A
Explanation
Both the statements are correct .
National Human Rights Commission
The National Human Rights Commission is a statutory (and not a constitutional) body. It was established in
1993 under a legislation enacted by the Parliament, namely, the Protection of Human Rights Act,1993. This Act
was amended in 2006.The commission is the watchdog of human rights in the country, that is, the rights
relating to life, liberty, equality and dignity of the individual guaranteed by the Constitution or embodied in
the international covenants and enforceable by courts in India.
The specific objectives of the establishment of the commission are :
(a) To strengthen the institutional arrangements through which human rights issues could be addressed in
their entirety in a more focused manner;
(b) To look into allegations of excesses, independently of the government, in a manner that would underline
the government's commitment to protect human rights; and
(c) To complement and strengthen the efforts that have already been made in this direction.

Question 66
Who among the following is not included in the selection committee that recommends appointment of the
State Information Commissioner (CIC) and other members?
A . Chief Minister
B . Leader of Opposition in Legislative Assembly
C . Speaker of Legislative Assembly

P a g e 70 | 100
www.learnmaze.com INDIAN POLITY – TEST 1 Learn Maze

D . State Cabinet Minister nominated by Chief Minister


Answer : C
Explanation
The Commission consists of a State Chief Information Commissioner and not more than ten State Information
Commissioners. They are appointed by the Governor on the recommendation of a committee consisting of the
Chief Minister as Chairperson, the Leader of Opposition in the Legislative Assembly and a State Cabinet
Minister nominated by the Chief Minister.

Question 67
In which of the following matters the power and status of State Legislative Council is equal to that of State
Legislative Assembly .
1 . Enlargement of jurisdiction of State Public Service Commission.
2 . Approval of ordinances issued by the governor.
3 . The council can vote on demand for grants .
Select the correct answer using the codes given below:
A . 1 and 2 only
B . 1 and 3 only
C . 2 and 3 only
D . All of the above
Answer : A
Brief Explanation
Statement 1 and 2 is correct .
Statement 2 is incorrect. The council can only discuss the budget but cannot vote on the demands for grants .
Detailed Explanation
Equal with Assembly
In the following matters, the powers and status of the council are broadly equal to that of the assembly:
1. Introduction and passage of ordinary bills. However, in case of disagreement between the two Houses, the
will of the assembly prevails over that of the council.

P a g e 71 | 100
www.learnmaze.com INDIAN POLITY – TEST 1 Learn Maze

2. Approval of ordinances issued by the governor.


3. Selection of ministers including the chief minister. Under the Constitution the, ministers including the chief
minister can be members of either House of the state legislature. However, irrespective of their membership,
they are responsible only to the assembly.
4. Consideration of the reports of the constitutional bodies like State Finance Commission, state public service
commission and Comptroller and Auditor General of India.
5. Enlargement of the jurisdiction of the state public service commission.

Question 68

Which of the following subjects are enlisted in Concurrent List.


1 . Organization of the High Courts
2 . Public Health and Sanitation
3 . Electricity
4 . Forests
Select the correct answer using the codes given below:
A . 1, 2 and 3 only
B . 1 and 3 only
C . 2 and 4 only
D . 3 and 4 only
Answer : D
Explanation
Subject 1 is enlisted in Union List
Subject 2 is enlisted in State List
Subject 3 and 4 is enlisted in Concurrent List .
Complete List of subjects
https://en.wikipedia.org/wiki/Concurrent_List

P a g e 72 | 100
www.learnmaze.com INDIAN POLITY – TEST 1 Learn Maze

https://en.wikipedia.org/wiki/Union_List
https://en.wikipedia.org/wiki/State_List

Question 69
Abolition of Legislative Council in States require which of the following bills .
A . Ordinary bill with simple majority of Parliament.
B . Constitution Amendment Bill with simple majority of Parliament.
C . Constitution Amendment Bill with special majority of Parliament.
D . Constitution Amendment Bill with special majority of Parliament and consent of half of the states.
Answer : B
Explanation
Abolition of Legislative Council in States require Constitution Amendment Bill with simple majority of
Parliament.
By Simple Majority of Parliament
A number of provisions in the Constitution can be amended by a simple majority of the two Houses of
Parliament outside the scope of Article 368.
These provisions include:
1. Admission or establishment of new states.
2. Formation of new states and alteration of areas, boundaries or names of existing states.
3. Abolition or creation of legislative councils in states.
4. Second Schedule—emoluments, allowances, privileges and so on of the president, the governors, the
Speakers, judges, etc.
5. Quorum in Parliament.
6. Salaries and allowances of the members of Parliament.
7. Rules of procedure in Parliament.
8. Privileges of the Parliament, its members and its committees.
9. Use of English language in Parliament.

P a g e 73 | 100
www.learnmaze.com INDIAN POLITY – TEST 1 Learn Maze

10. Number of puisne judges in the Supreme Court.


11. Conferment of more jurisdiction on the Supreme Court.
12. Use of official language.
13. Citizenship—acquisition and termination.
14. Elections to Parliament and state legislatures.
15. Delimitation of constituencies.
16. Union territories.
17. Fifth Schedule—administration of scheduled areas and scheduled tribes.
18. Sixth Schedule—administration of tribal areas.

Question 70
The Preamble of India states that India is a republic country. Why is India referred as a republic ?
1 . Vesting of political sovereignty in the President of India
2 . Absence of privileged class
3 . India has an elected head called the President .
Which of the above is/are correct?
A . 1 and 2 only
B . 1 and 3 only
C . 2 and 3 only
D . 3 only
Answer : C
Explanation
Statement 1 is incorrect . Political sovereignty is vested in the people and not in the President .
Statement 2 is correct . Sovereignty means the absence of any privileged class and hence all public offices
being opened to every citizen without any discrimination.
Statement 3 is correct . In a monarchy, the head of the state (usually king or queen) enjoys a hereditary
position, that is, he comes into office through succession, eg, Britain. In a republic, on the other hand, the

P a g e 74 | 100
www.learnmaze.com INDIAN POLITY – TEST 1 Learn Maze

head of the state is always elected directly or indirectly for a fixed period, eg, USA. Therefore, the term
‘republic’ in our Preamble indicates that India has an elected head called the president.

Question 71
Which of the following case judgment attempted to curb blatant misuse of Article 356 of the Constitution of
India ?
A . S. R. Bommai v. Union of India
B . IC Golaknath v State of Punjab
C . Minerva Mills v Union of India
D . Keshavananda Bharti vs State of Kerala
Answer : A
Explanation
Option A is correct . S. R. Bommai v. Union of India was a landmark judgment of the Supreme Court of India,
where the Court discussed at length provisions of Article 356 of the Constitution of India and related issues.
This case had huge impact on Centre-State Relations. The judgement attempted to curb blatant misuse of
Article 356 of the Constitution of India, which allowed President's rule to be imposed over state governments .

Question 72
Consider the following statements in regard with ADM Jabalpur v Shivakant Shukla Case .
1 . Supreme Court applied doctrine of procedure established by law in this case
2 . This case is also known as Habeas Corpus case
Which of the above is/are correct?
A. 1 only
B. 2 only
C. Both 1 and 2
D. None of the above
Answer : C
Brief Explanation
P a g e 75 | 100
www.learnmaze.com INDIAN POLITY – TEST 1 Learn Maze

Statement 1 is correct . Supreme Court applied doctrine of procedure established by law in ADM Jabalpur v
Shivakant Shukla Case . the Supreme Court declared that the rights of citizens to move the court for violation
of Articles 14, 21 and 22 would remain suspended during emergencies.
Statement 2 is also correct .
Detailed Explanation
While the High court judgment was appealed to SC, Indira Gandhi, faced by an unprecedented protest from
an opposition united under J P Naryanan, invoked article 352 declaring National emergency on the grounds of
threat from Internal disturbance.
Censorship muzzled the Press, the opposition was silenced and the common man terrorized. The government
made extensive use of preventive detention, arresting people not because they have committed any offence,
but on the apprehension that they may commit one.
Many cases were filed in the courts against it and 9 High Courts gave judgments that even during emergency
the courts could entertain a writ of habeas corpusfiled by a person challenging his/her detention
The Government (read Indira Gandhi) decided to appeal against these decisions to the Supreme Court. It was
thus that the Constitutional bench of five Judges came to be constituted to hear case, dubbed as A.D.M.
Jabalpur vs. Shukla.
Supreme Court applied doctrine of procedure established by law in letter but not in spirit and overturned the
judgement by high courts, declaring that article 32 –the right to approach to court to defend fundamental
rights- remains suspended under emergency.

Question 73
Central Bureau of Investigation functioning under which of the following ministry.
A . Ministry of Home Affairs
B . Ministry of Social Justice & Empowerment
C . Ministry of Law and Justice
D . Ministry of Personnel
Answer : D
Explanation
The Central Bureau of Investigation (CBI ) is functioning under Deptt. of Personnel, Ministry of Personnel,
Pension & Public Grievances.

Question 74
P a g e 76 | 100
www.learnmaze.com INDIAN POLITY – TEST 1 Learn Maze

Which of the following are the features of Lokpal and Lokayuktas Act 2013 ?
1 . Lokpal can suo motu proceed against any public servant.
2 . The Lokpal will have the power of superintendence and direction over any investigating agency, including
the CBI, for cases referred to them by the Lokpal.
3 . Anonymous complaints is allowed if there are proper supporting documents .
Which of the above is/are correct?
A . 1 and 2 only
B . 1 and 3 only
C . 2 only
D . 3 only
Answer : C
Brief Explanation
Statement 1 is incorrect . Lokpal cannot suo motu proceed against any public servant.
Statement 2 is correct . It is one of the greatest feature of Lokpal and Lokayuktas Act
Statement 3 is incorrect . Anonymous complaints are not allowed – Can’t just make a complaint on plain paper
and drop it in a box with supporting documents.
Detailed Explanation
The salient features of the Lokpal and Lokayuktas Act (2013) are as follows.
1. It seeks to establish the institution of the Lokpal at the Centre and the Lokayukta at the level of the State
and thus seeks to provide a uniform vigilance and anti-corruption road map for the nation both at the Centre
and at the States. The jurisdiction of Lokpal includes the Prime Minister, Ministers, Members of Parliament
and Groups A, B, C and D officers and officials of the Central Government.
2. The Lokpal to consist of a Chairperson with a maximum of 8 members of which 50% shall be judicial
members.
3. 50% of the members of the Lokpal shall come from amongst the SCs, the STs, the OBCs, minorities and
women.
4. The selection of the Chairperson and the members of Lokpal shall be through a Selection Committee
consisting of the Prime Minister, the Speaker of the Lok Sabha, the Leader of the Opposition in the Lok Sabha,
the Chief Justice of India or a sitting Supreme Court Judge nominated by the Chief Justice of India and an
eminent jurist to be nominated by the President of India on the basis of recommendations of the first four
members of the selection committee.
P a g e 77 | 100
www.learnmaze.com INDIAN POLITY – TEST 1 Learn Maze

5. A Search Committee will assist the Selection Committee in the process of selection. 50% of the members of
the Search Committee shall also be from amongst the SCs, the STs, the OBCs, minorities and women.
6. The Prime Minister has been brought under the purview of the Lokpal with subject matter exclusions and
specific process for handling complaints against the Prime Minister.
7. Lokpal’s jurisdiction will cover all categories of public servants, including Group A, Group B, Group C, and
Group D officers and employees of Government. On complaints referred to the CVC by the Lokpal, the CVC will
send its report of preliminary enquiry in respect of Group A and Group B Officers back to the Lokpal for further
decision. With respect to categories of employees from Group C and Group D, the CVC will proceed further in
exercise of its own powers under the CVC Act subject to reporting and review by the Lokpal.
8. The Lokpal will have the power of superintendence and direction over any investigating agency, including
the CBI, for cases referred to them by the Lokpal.
9. A High-Powered Committee chaired by the Prime Minister will recommend the selection of the Director of
CBI.
10. It incorporates provisions for attachment and confiscation of property of public servants acquired by
corrupt means, even while the prosecution is pending.
11. It lays down clear timelines. For preliminary enquiry, it is three months extendable by three months. For
investigation, it is six months which may be extended by six months at a time. For trial, it is one year
extendable by one year and to achieve this, special courts to be set up.
12. It enhances maximum punishment under the Prevention of Corruption Act from seven years to ten years.
The minimum punishment under sections 7, 8, 9 and 12 of the Prevention of Corruption Act will now be three
years, and the minimum punishment under section 15 (punishment for attempt) will now be two years.
13. Institutions which are financed fully or partly by Government are under the jurisdiction of Lokpal, but
institutions aided by Government are excluded.
14. It provides adequate protection for honest and upright public servants.
15. Lokpal conferred with power to grant sanction for prosecution of public servants in place of the
Government or competent authority.
16. It contains a number of provisions aimed at strengthening the CBI such as:
(i) setting up of a Directorate of Prosecution headed by a Director of Prosecution under the overall control of
the Director of CBI;
(ii) appointment of the Director of Prosecution on the recommendation of the CVC;
(iii) maintenance of a panel of advocates by CBI other than Government advocates with the consent of the
Lokpal for handling Lokpal-referred cases;
(iv) transfer of officers of CBI investigating cases referred by Lokpal with the approval of Lokpal;
P a g e 78 | 100
www.learnmaze.com INDIAN POLITY – TEST 1 Learn Maze

(v) provision of adequate funds to CBI for investigating cases referred by Lokpal.
17. All entities receiving donations from foreign source in the context of the Foreign Contribution Regulation
Act (FCRA) in excess of Rs.10 lakhs per year are brought under the jurisdiction of Lokpal.
18. It contains a mandate for setting up of the institution of Lokayukta through enactment of a law by the
State Legislature within a period of 365 days from the date of commencement of this Act. Thus, the Act
provides freedom to the states to decide upon the contours of the Lokayukta mechanism in their respective
states.

Question 75
Identify the executive powers of President of India from the following statements:
1 . He directly administers the union territories through administrators appointed by him .
2 . He appoint the Chief Justice and the judges of Supreme Court and high courts.
3 . No demand for a grant can be made except on his recommendation.
4 . He can declare any area as scheduled area.
Select the correct answer using the codes given below:
A . 1, 2 and 4 only
B . 1 , 3 and 4 only
C . 1 and 4 only
D . 2 and 4 only
Answer : C
Explanation
Option 1 is correct . It is the executive power of the President
Option 2 is incorrect. Appointment of Chief Justice and the judges of Supreme Court and high courts is judicial
power of the President .
Option 3 is incorrect. This is a financial power of the President
Option 4 is correct. President can declare any area as scheduled area and has powers with respect to the
administration of scheduled areas and tribal areas.

P a g e 79 | 100
www.learnmaze.com INDIAN POLITY – TEST 1 Learn Maze

Question 76
Consider the following statements
Assertion (A) : 91st Amendment Act aimed at limiting the number of Council of Ministers
Reason (R) : Coalition governments in India had Jumbo cabinet .
In the context of the above, which of these is correct?
A . A is correct, and R is a correct explanation of A.
B . A is correct, but R is not a correct explanation of A.
C . A is correct, but R is incorrect.
D . Both A and R are incorrect.
Answer : A
Explanation
Both the Assertion and Reason is correct . On July 7, the 91st Amendment to the Constitution, limiting the size
of the Council of Ministers at the Centre and the States to no more than 15 per cent of the numbers in the Lok
Sabha or the State Legislature, came into effect. Previous govts. had jumbo cabinets to satisfy the coalition
partners .

Question 77
Which of the following High Courts has more one state /UT under its jurisdiction ?
1 . Kerala High Court
2 . Gauhati High Court
3 . Madras High Court
4 . Bombay High Court
5 . Madhya Pradesh High Court
Select the correct answer using the codes given below:
A . 1, 2, 3 and 4 only
B . 1, 3 , 4 and 5 only
C . 2, 3 , 4 and 5 only

P a g e 80 | 100
www.learnmaze.com INDIAN POLITY – TEST 1 Learn Maze

D . 2 , 3 and 4 only
Answer : A
Explanation
Option 1 . Kerala High Court - Kerala, Lakshadweep
Option 2 . Gauhati High Court - Arunachal Pradesh, Assam, Mizoram, Nagaland
Option 3 . Madras High Court - Pondicherry, Tamil Nadu
Option 4 . Bombay High Court - Goa, Dadra and Nagar Haveli, Daman and Diu, Maharashtra
Option 5 . Madhya Pradesh High Court - Madhya Pradesh

Question 78
Consider the following statements in regard with the governor.
1 . The draft constitution provided for direct election of the governor on the basis of universal adult suffrage.
2 . The oath of the governor is administered by the Chief Justice of High Court.
3 . He appoints advocate general of the state.
Select the correct answer using the codes given below:
A . 1 and 2 only
B . 1 and 3 only
C . 2 and 3 only
D . All of the above
Answer : D
Brief Explanation
All the 3 statements are correct .
Detailed Explanation
Statement 1
The Draft Constitution provided for the direct election of the governor on the basis of universal adult suffrage.
But the Constituent Assembly opted for the present system of appointment of governor by the president
because of the following reasons1:

P a g e 81 | 100
www.learnmaze.com INDIAN POLITY – TEST 1 Learn Maze

1. The direct election of the governor is incompatible with the parliamentary system established in the states.
2. The mode of direct election is more likely to create conflicts between the governor and the chief minister.
3. The governor being only a constitutional (nominal) head, there is no point in making elaborate
arrangements for his election and spending huge amount of money.
4. The election of a governor would be entirely on personal issues. Hence, it is not in the national interest to
involve a large number of voters in such an election.
5. An elected governor would naturally belong to a party and would not be a neutral person and an impartial
head.
6. The election of governor would create separatist tendencies and thus affect the political stability and unity
of the country.
7. The system of presidential nomination enables the Centre to maintain its control over the states.
8. The direct election of the governor creates a serious problem of leadership at the time of a general election
in the state.
9. The chief minister would like his nominee to contest for governorship.
Statement 2
The oath of office to the governor is administered by the chief justice ofthe concerned state high court and in
his absence, the senior-most judge of that court available. Every person discharging the functions of the
governor also undertakes the similar oath or affirmation.
Statement 3
He appoints the advocate general of a state and determines his remuneration. The advocate general holds
office during the pleasure of the governor.

Question 79
Consider the following statements regarding All India Judicial Services.
1 . The 44th amendment Act of 1978 made the provision for creation of All India Judicial Services .
2 . Parliament can create AIJS if Lok Sabha passes a resolution declaring that it is necessary with the support
of 2/3rd of the members present and voting .
Which of the above is/are correct?
A. 1 only
B. 2 only
C. Both 1 and 2
P a g e 82 | 100
www.learnmaze.com INDIAN POLITY – TEST 1 Learn Maze

D. None of the above


Answer : D
Explanation
Statement 1 is incorrect . The 42nd amendment Act of 1976 made the provision for creation of All India
Judicial Services .
Statement 2 is incorrect . To protect the interests of federal system the power of recommendation is given to
Rajya Sabha and not Lok Sabha . Such a resolution can be passed only in Rajya Sabha .

Question 80
Which of the following statements is correct regarding Elections Commission in India ?
1 . The election of President , Vice President and Lok Sabha speaker is conducted by Elections Commission .
2 . The Commission also has advisory jurisdiction in the matter of post election disqualification of sitting
members of Parliament and State Legislatures.
Which of the above is/are correct?
A. 1 only
B. 2 only
C. Both 1 and 2
D. None of the above
Answer : B
Explanation
Statement 1 is incorrect . The power of supertendence, direction and conduct of elections to the Parliament,
the state legislatures, the office of the President and the office of the Vice-President is vested in the
Commission.
Statement 2 is correct .
Advisory Jurisdiction & Quasi-Judicial Functions
Under the Constitution, the Commission also has advisory jurisdiction in the matter of post election
disqualification of sitting members of Parliament and State Legislatures. Further, the cases of persons found
guilty of corrupt practices at elections which come before the Supreme Court and High Courts are also
referred to the Commission for its opinion on the question as to whether such person shall be disqualified
and, if so, for what period. The opinion of the Commission in all such matters is binding on the President or, as
the case may be, the Governor to whom such opinion is tendered.

P a g e 83 | 100
www.learnmaze.com INDIAN POLITY – TEST 1 Learn Maze

The Commission has the power to disqualify a candidate who has failed to lodge an account of his election
expenses within the time and in the manner prescribed by law. The Commission has also the power for
removing or reducing the period of such disqualification as also other disqualification under the law.
However, when there is some dispute regarding the election process of the elected MPs, then it is the High
Court that decides.

Question 81
Consider the following statements in the context of Fundamental rights.
1 . Article 18 prohibits a citizen of India from accepting any titles from any foreign state .
2 . The State cannot impose reasonable restrictions on exercise of freedom of speech and expression ..
3 . Right to strike , picketing and demonstration is included in freedom of expression .
Select the correct answer using the codes given below:
A . 1 and 2 only
B . 1 and 3 only
C . 1 only
D . 3 only
Answer : C
Explanation
Statement 1 is correct . Article 18 abolishes titles and prohibits a citizen of India from accepting any titles from
any foreign state .
Statement 2 is incorrect . The State can impose reasonable restrictions on the exercise of the freedom of
speech and expression on the grounds of sovereignty and integrity of India, security of the state, friendly
relations with foreign states, public order, decency or morality, contempt of court, defamation, and
incitement to an offence.
Statement 3 is incorrect. Freedom of speech and expression provides right to demonstration and picketing but
not right to strike.

Question 82
The system of government is varied in different countries. The “Head of the Government” and the related
country is provided. Find out which of the following position is incorrectly matched.
A . Chancellor - Germany

P a g e 84 | 100
www.learnmaze.com INDIAN POLITY – TEST 1 Learn Maze

B . Chief Executive - Afghanistan


C . Prime Minister - Brazil
D . President – South Africa
Answer : C
Explanation
Option C is incorrect. In Brazil the Head of the State and Head of the government is the President and not the
Prime Minister.

Question 83
In which among the following Union Territories the administrator appointed by President is called as
Lieutenant Governor ?

1 . Andaman and Nicobar Island


2 . Chandigarh
3 . Delhi
4 . Lakshadweep
5 . Puducherry
Select the correct answer using the codes given below:
A . 1, 2, 3 and 4 only
B . 1, 3 and 5 only
C . 2, 3 , 4 and 5 only
D . 3 , 4 and 5 only
Answer : B
Explanation

P a g e 85 | 100
www.learnmaze.com INDIAN POLITY – TEST 1 Learn Maze

Question 84
In regard to Contingency Fund of India, consider the following statements :
1 . It is placed at the disposal of the President to enable him/her to make advances to meet urgent unforeseen
expenditure, pending authorization by the Parliament.
2 . The fund is held by the Finance Secretary (Department of Economic Affairs) on behalf of the President of
India .
Which of the above is/are correct?
A. 1 only
B. 2 only
C. Both 1 and 2
D. None of the above
Answer : C
Explanation
Statement 1 is correct . The Contingency Fund of India established under Article 267 (1) of the Constitution is
in the nature of an imprest (money maintained for a specific purpose) which is placed at the disposal of the
President to enable him/her to make advances to meet urgent unforeseen expenditure, pending authorization
by the Parliament. Approval of the legislature for such expenditure and for withdrawal of an equivalent
amount from the Consolidated Fund is subsequently obtained to ensure that the corpus of the Contingency
Fund remains intact. The corpus for Union Government at present is Rs 500 crore (Rs 5 billion) and is
enhanced from time to time by the Union Legislature.
P a g e 86 | 100
www.learnmaze.com INDIAN POLITY – TEST 1 Learn Maze

Statement 2 is correct . The Ministry of Finance operates this Fund on behalf of the President of India. The
fund is held by the Finance Secretary (Department of Economic Affairs) on behalf of the President of India and
it can be operated by executive action.

Question 85
Which of the following is incorrect regarding Government of India Act 1919
A . It required that the three of the six members of the Viceroy’s executive Council were to be Indian.
B . It granted franchise to a limited number of people on the basis of property, tax or education.
C . It separated, for the first time, provincial budgets from the Central budget and authorised the provincial
legislatures to enact their budgets.
D . It introduced a system of communal representation for Muslims by accepting the concept of ‘separate
electorate’.
Answer : D
Brief Explanation
Option A ,B and C are features of Government of India Act 1919 .
Option D is a feature of Morley – Minto Reforms
Detailed Explanation
Features of Government of India Act 1919
1. It relaxed the central control over the provinces by demarcating and separating the central and provincial
subjects. The central and provincial legislatures were authorised to make laws on their respective list of
subjects. However, the structure of government continued to be centralised and unitary.

2. It further divided the provincial subjects into two parts—transferred and reserved. The transferred subjects
were to be administered by the governor with the aid of ministers responsible to the legislative Council.
The reserved subjects, on the other hand, were to be administered by the governor and his executive council
without being responsible to the legislative Council. This dual scheme of governance was known as‘dyarchy
’—a term derived from the Greek word di-arche which means double rule. However, this experiment was
largely unsuccessful.
3. It introduced, for the first time, bicameralism and direct elections in the country. Thus, the Indian Legislative
Council was replaced by a bicameral legislature consisting of an Upper House (Council of State) and a Lower
House (Legislative Assembly). The majority of members of both the Houses were chosen by direct election.

4. It required that the three of the six members of the Viceroy’s executive Council (other than the
commander-in-chief) were to be Indian.
P a g e 87 | 100
www.learnmaze.com INDIAN POLITY – TEST 1 Learn Maze

5. It extended the principle of communal representation by providing separate electorates for Sikhs, Indian
Christians, Anglo-Indians and Europeans.
6. It granted franchise to a limited number of people on the basis of property, tax or education.
7. It created a new office of the High Commissioner for India in London and transferred to him some of the
functions hitherto performed by the Secretary of State for India.
8. It provided for the establishment of a public service commission. Hence, a Central Public Service
Commission was set up in 1926 for recruiting civil servants.
9. It separated, for the first time, provincial budgets from the Central budget and authorised the provincial
legislatures to enact their budgets.
10. It provided for the appointment of a statutory commission to inquire into and report on its working after
ten years of its coming into force.

Question 86
Which among the following constitute electoral offences under the Representation of the People Act, 1951?
1 . Suppression of election expenses incurred
3 . Publishing results of Exit Polls during elections.

Which of the above is/are correct?


A. 1 only
B. 2 only
C. Both 1 and 2
D. None of the above
Answer : C
Explanation
Statement 1 is correct . Umlesh Yadav is the first politician to be disqualified by the Election Commission of
India for a period of three years for suppression of her election expenses incurred when she was elected as an
MLA to the Bisauli constituency in the Uttar Pradesh state assembly elections, 2007.
Statement 2 is correct . No person shall conduct any exit poll and publish or publicise by means of the print or
electronic media or disseminate in any other manner, whatsoever, the result of any exit poll during such
period, as may be notified by the Election Commission in this regard.

Question 87
P a g e 88 | 100
www.learnmaze.com INDIAN POLITY – TEST 1 Learn Maze

Which of the following case judgments shook the conservative religious laws in India ?
1 . Lily Thomas v. Union of India case
2 . Mohd. Ahmed Khan v. Shah Bano Begum
3 . Mary Roy v. State Of Kerala
Select the correct answer using the codes given below:
A . 1 and 2 only
B . 2 and 3 only
C . 2 only
D . 3 only
Answer : B
Explanation
Option 1 is incorrect . Lily Thomas v. Union of India case - Supreme Court of India ruled that any Member of
Parliament (MP), Member of the Legislative Assembly (MLA) or Member of a Legislative Council (MLC) who is
convicted of a crime and awarded a minimum of two year imprisonment, loses membership of the House with
immediate effect .
Option 2 is correct . Mohd. Ahmed Khan v. Shah Bano Begum - The petitioner challenged the Muslim personal
law. The Supreme Court ruled in favour of Shah Bano and granted her alimony. Most favoured it as a secular
judgment but it also invoked a strong reaction from the Muslim community, which felt that the judgment was
an encroachment on Muslim Sharia law and hence led to the formation of the All India Muslim Personal Law
Board in 1973.
Option 3 is correct . Mary Roy v. State Of Kerala - Mary Roy was denied her share of the familial property due
to the sexist Travancore Succession Act of 1916. She sued her brothers after her father's death. This was the
case that made its way through the Indian court system and which she won.
It was in 1986 that Ms. Roy had obtained a landmark judgment from the Supreme Court entitling Syrian
Christian women to an equal share in their father's property. Till then, the Syrian Christian community
followed the provisions of the Travancore Succession Act, 1916 and Cochin Succession Act, 1921 even as
members of the community in other parts of the country were governed by the provisions of the Indian
Succession Act, 1925. As per the Travancore and the Cochin Acts, daughters were eligible for one quarter of
the sons' share or Rs.5,000, which ever was less, if the father died intestate.

Question 88
P a g e 89 | 100
www.learnmaze.com INDIAN POLITY – TEST 1 Learn Maze

In which of the following the President of India has made consultative jurisdiction
1 . Ram Janma Bhumi Case 1993
2 . Kerala Education Bill 1958
3 . Legal Services Authorities Act 1987
4 . Punjab Termination of Agreements Act 2004
5 . Indecent Representation of Women (Prohibition) Act 1986
Select the correct answer using the codes given below:
A . 1 , 2 and 4 only
B . 2 ,3 , 4 and 5 only
C . 1 ,3 , 4 and 5 only
D . 1 , 4 and 5 only
Answer : A
Explanation
So far (2013), the President has made fifteen references to the Supreme Court under its advisory jurisdiction
(also known as consultative jurisdiction). These are mentioned below in the chronological order.
1. Delhi Laws Act in 1951
2. Kerala Education Bill in 1958
3. Berubari Union in 1960
4. Sea Customs Act in 1963
5. Keshav Singh’s case relating to the privileges of the Legislature in 1964
6. Presidential Election in 1974
7. Special Courts Bill in 1978
8. Jammu and Kashmir Resettlement Act in 1982
9. Cauvery Water Disputes Tribunal in 1992
10. Rama Janma Bhumi case in 1993
11. Consultation process to be adopted by the chief justice of India in 1998
12. Legislative competence of the Centre and States on the subject of natural gas and liquefied natural gas in
2001
P a g e 90 | 100
www.learnmaze.com INDIAN POLITY – TEST 1 Learn Maze

13. The constitutional validity of the Election Commission’s decision on deferring the Gujarat Assembly
Elections in 2002
14. Punjab Termination of Agreements Act in 2004
15. 2G spectrum case verdict and the mandatory auctioning of natural resources across all sectors in 2012

Question 89
Consider the following statements regarding co-operative societies.
1 . Part IX –B contains provisions of co-operative societies
2 . Co-operative societies in India enjoy constitutional status.
3 . Right to form co-operative societies is a legal right
Which of the above is/are correct?
A . 1 and 2 only
B . 1 and 3 only
C . 2 and 3 only
D . All of the above
Answer : A
Explanation
Statement 1 is correct. Part IX-B deals with co-operative societies .
Statement 2 is correct. The 97th constitutional amendment act of 2011 gave a constitutional status and
protection to co-operative societies.
Statement 3 is incorrect. The right to form co-operative societies is a fundamental right and not a legal right .

Question 90
Which of the following urban local bodies are created in India for the administration of urban areas .
1 . Special Purpose Agency
2 . Port Trust
3 . Township
4 . Notified Area Committee
Select the correct answer using the codes given below.
P a g e 91 | 100
www.learnmaze.com INDIAN POLITY – TEST 1 Learn Maze

A . 1, 2 and 3 only
B . 1 and 3 only
C . 1, 2 and 4 only
D . All of the above
Answer : D
Explanation
All the above options are correct.
The following eight types of urban local bodies are created in India for the administration of urban areas:

 Municipal Corporation
 Municipality
 Notified Area Committee
 Town Area Committee
 Cantonment Board
 Township
 Port Trust
 Special Purpose Agency

Question 91
Which of the following is true regarding removal of judges of High Court ?
1 . Removal motion must be signed by 50 members of Lok Sabha and is to be given to the Speaker
2 . The Judges (Protection) Act, 1985 regulates the procedure relating to removal of a judge of a High Court .
Which of the above is/are correct?
A. 1 only
B. 2 only
C. Both 1 and 2
D. None of the above
Answer : D
Brief Explanation
P a g e 92 | 100
www.learnmaze.com INDIAN POLITY – TEST 1 Learn Maze

Statement 1 is incorrect . A removal motion signed by 100 members (in the case of Lok Sabha) or 50 members
(in the case of Rajya Sabha) is to be given to the Speaker/Chairman.
Statement 2 is incorrect. The Judges Enquiry Act (1968) regulates the procedure relating to the removal of a
judge of a high court by the process of impeachment.
Detailed Explanation
Removal of Judges A judge of a high court can be removed from his office by an order of the President. The
President can issue the removal order only after an address by the Parliament has been presented to him in
the same session for such removal. The address must be supported by a special majority of each House of
Parliament (i.e., a majority of the total membership of that House and majority of not less than two-thirds of
the members of that House present and voting). The grounds of removal are two—proved misbehaviour or
incapacity. Thus, a judge of a high court can be removed in the same manner and on the same grounds as a
judge of the Supreme Court.
The Judges Enquiry Act (1968) regulates the procedure relating to the removal of a judge of a high court by the
process of impeachment:
1. A removal motion signed by 100 members (in the case of Lok Sabha) or 50 members (in the case of Rajya
Sabha) is to be given to the Speaker/Chairman.
2. The Speaker/Chairman may admit the motion or refuse to admit it.
3. If it is admitted, then the Speaker/Chairman is to constitute a three member committee to investigate into
the charges.
4. The committee should consist of (a) the chief justice or a judge of the Supreme Court, (b) a chief justice of a
high court, and (c) a distinguished jurist.
5. If the committee finds the judge to be guilty of misbehaviour or suffering from an incapacity, the House can
take up the consideration of the motion.
6. After the motion is passed by each House of Parliament by special majority, an address is presented to the
president for removal of the judge.
7. Finally, the president passes an order removing the judge.

Question 92
In regard with Anti defection law which of the following amount to disqualification.
1 . An independent member of a House becomes disqualified to remain a member of the House if he joins any
political party after such election.

P a g e 93 | 100
www.learnmaze.com INDIAN POLITY – TEST 1 Learn Maze

2 . If a member, after being elected as the presiding officer of the House, voluntarily gives up the membership
of his party or rejoins it after he ceases to hold that office.
3 . A nominated member of a House join any political party within six months of taking his seat in the House.
Select the correct answer using the codes given below.
A . 1 and 2 only
B . 1 and 3 only
C . 2 and 3 only
D . 1 only
Answer : D
Explanation
Statement 1 is correct . Independent Members: An independent member of a House (elected without being
set up as a candidate by any political party) becomes disqualified to remain a member of the House if he joins
any political party after such election.
Statement 2 is incorrect .
The disqualification on the ground of defection does not apply in the following two cases:
(a) If a member goes out of his party as a result of a merger of the party with another party. A merger takes
place when two-thirds of the members of the party have agreed to such merger.
(b) If a member, after being elected as the presiding officer of the House, voluntarily gives up the membership
of his party or rejoins it after he ceases to hold that office. This exemption has been provided in view of the
dignity and impartiality of this office.
Statement 3 is incorrect . Nominated Members: A nominated member of a House becomes disqualified for
being a member of the House if he joins any political party after the expiry of six months from the date on
which he takes his seat in the House. This means that he may join any political party within six months of
taking his seat in the House without inviting this disqualification.

Question 93
In regard with the composition of Parliament consider the following statements .
1 . At present thirteen Lok Sabha members are elected from Union Territories .
2 . Fifth schedule of the constitution deals with the allocation of seats in Rajya Sabha
3 . All States are provided equal representation irrespective of their population in Rajya Sabha
Select the correct answer using the codes given below.
P a g e 94 | 100
www.learnmaze.com INDIAN POLITY – TEST 1 Learn Maze

A . 1 and 2 only
B . 1 only
C . 2 only
D . All of the above
Answer : B
Explanation
Statement 1 is correct.

Statement 2 is incorrect. Fourth schedule of the constitution deals with the allocation of seats in Rajya Sabha
Statement 3 is incorrect. All States are provided equal representation irrespective of their population in
Senate in U.S.A . In India R.S. seats are allocated on the basis of population .

Question 94
Which of the following statements are correct regarding Constituent Assembly ?
1 . It ratified Indias membership of the Commonwealth .
2 . Union Constitution Committee was chaired by B.R. Ambedkar
Which of the above is/are correct?
A. 1 only
B. 2 only
C. Both 1 and 2
D. None of the above
Answer : A
Explanation
P a g e 95 | 100
www.learnmaze.com INDIAN POLITY – TEST 1 Learn Maze

Statement 1 is correct. Constituent Assembly ratified India’s membership of the Commonwealth in 1949 .
Statement 2 is incorrect. Union Constitution Committee was chaired by Jawaharlal Nehru .

Question 95
Original Jurisdiction of Supreme Court of India includes which of the following.
A . Recovery of damages by a state against the Centre
B . Dispute between two or more states
C . Recovery of damages by a state against the Centre .
D . A dispute arising out of any pre-Constitution treaty or agreement
Answer : B
Explanation
As a federal court, the Supreme Court under original jurisdiction decides the disputes between different units
of the Indian Federation. More elaborately, any dispute between:
(a) the Centre and one or more states; or
(b) the Centre and any state or states on one side and one or more states on the other; or
(c) between two or more states.
In the above federal disputes, the Supreme Court has exclusive original jurisdiction. Exclusive means, no other
court can decide such disputes and original means, the power to hear such disputes in the first instance, not
by way of appeal.
Original jurisdiction of the Supreme Court does not extend to the following:
(a) A dispute arising out of any pre-Constitution treaty, agreement, covenant, engagement, sanad or other
similar instrument.
(b) A dispute arising out of any treaty, agreement, etc., which specifically provides that the said jurisdiction
does not extent to such a dispute.9
(c) Inter-state water disputes.10
(d) Matters referred to the Finance Commission.
(e) Adjustment of certain expenses and pensions between the Centre and the states.
(f) Ordinary dispute of Commercial nature between the Centre and the states.

P a g e 96 | 100
www.learnmaze.com INDIAN POLITY – TEST 1 Learn Maze

(g) Recovery of damages by a state against the Centre.

Question 96
Which among the following statements is not true regarding Consumer Protection Act 1986 ?
A . Consumer has the right to seek redressal against unfair trade practices.
B . Complaint is to be filed within 6 months of buying the product or using the service.
C . Under the Consumer Protection Act, every district has at least one consumer redressal forum also called a
consumer court.
D . Claims of less than Rs. 5 lakh should be filed with district forum
Answer : B
Statement A , C and D are correct .
Statement B is incorrect . Complaint is to be filed within two years of buying the product or using the service.
Detailed Document - http://www.wipo.int/edocs/lexdocs/laws/en/in/in076en.pdf

Question 97
Consider the following statements regarding devices of parliamentary proceedings.
1 . Rajya Sabha is not permitted to make use of adjournment motion
2 . Zero Hour is mentioned in the Rules of Procedure.
Which of the above is/are correct?
A. 1 only
B. 2 only
C. Both 1 and 2
D. None of the above
Answer : A
Explanation

P a g e 97 | 100
www.learnmaze.com INDIAN POLITY – TEST 1 Learn Maze

Statement 1 is correct . It involves an element of censure against the government and hence Rajya Sabha is
not permitted to use this device .
Statement 2 is incorrect . Unlike the question hour, the zero hour is not mentioned in the Rules of Procedure.
Thus it is an informal device available to the members of the Parliament to raise matters without any prior
notice. The zero hour starts immediately after the question hour and lasts until the agenda for the day (ie,
regular business of the House) is taken up. In other words, the time gap between the question hour and the
agenda is known as zero hour. It is an Indian innovation in the field of parliamentary procedures and has been
in existence since 1962.

Question 98
Which of the following statements are true in regard with the Speaker of Lok Sabha ?
1 . Speaker is the final interpreter of the provisions of Constitution of India inside the House
2 . When Lok Sabha is dissolved speaker has to vacate his office
3 . Speaker of Lok Sabha derives his duties only from Rules of Procedure and Constitution of India
4 . The work and conduct of Speaker of Lok Sabha can be discussed and criticized on a substantive motion.
Select the correct answer using the codes given below:
A . 1, 2 and 4 only
B . 1 , 3 and 4 only
C . 1 and 4 only
D . 2 , 3 and 4 only
Answer : C
Explanation
Statement 1 is correct . He is the final interpreter of the provisions of (a) the Constitution of India, (b) the
Rules of Procedure and Conduct of Business of Lok Sabha, and (c) the parliamentary precedents, within the
House.
Statement 2 is incorrect . Whenever the Lok Sabha is dissolved, the Speaker does not vacate his office and
continues till the newly-elected Lok Sabha meets.
Statement 3 is incorrect . The Speaker of the Lok Sabha derives his powers and duties from three sources, that
is, the Constitution of India, the Rules of Procedure and Conduct of Business of Lok Sabha, and Parliamentary
Conventions (residuary powers that are unwritten or unspecified in the Rules).

P a g e 98 | 100
www.learnmaze.com INDIAN POLITY – TEST 1 Learn Maze

Statement 4 is correct . The work and conduct of Speaker cannot be discussed and criticized in the Lok Sabha
except on a substantive motion.

Question 99
Which of the following is correct regarding Public Interest Litigation ?
1 . Relaxation of traditional rule of locus standi
2 . Facilitating effective access to justice to the socially and economically weaker sections of the society .
Which of the above is/are correct?
A. 1 only
B. 2 only
C. Both 1 and 2
D. None of the above
Answer : C
Explanation
Statement 1 is correct . The introduction of PIL in India was facilitated by the relaxation of the traditional rule
of ‘locus standi’. According to this rule, only that person whose rights are infringed alone can move the court
for the remedies, whereas, the PIL is an exception to this traditional rule. Under the PIL, any public-spirited
citizen or a social organisation can move the court for the enforcement of the rights of any person or group of
persons who because of their poverty or ignorance or socially or economically disadvantaged position are
themselves unable to approach the court for the remedies. Thus, in a PIL, any member of the public having
‘sufficient interest’ can approach the court for enforcing the rights of other persons and redressal of a
common grievance.
Statement 2 is correct .
The real purposes of PIL are:
(i) vindication of the rule of law,
(ii) facilitating effective access to justice to the socially and economically weaker sections of the society, and
(iii) meaningful realization of the fundamental rights.

Question 100

P a g e 99 | 100
www.learnmaze.com INDIAN POLITY – TEST 1 Learn Maze

Urban government was constituted through 74th constitutional amendment of 1992. At the central level the
subject of urban local government is dealt by which of the following ministries.
1 . Ministry of Social Justice and Empowerment
2 . Ministry of Urban Development
3 . Ministry of Defence
4 . Ministry of Home Affairs
Select the correct answer using the codes given below:
A . 1, 2 and 4 only
B . 1 , 3 and 4 only
C . 1 and 4 only
D . 2 , 3 and 4 only
Answer : D
Explanation
Option 1 is incorrect.
Option 2 is correct. Ministry of Urban Development created as a separate ministry in 1985 .
Option 3 is correct. Ministry of Defence in the case of cantonment boards
Option 4 is correct. Ministry of Home Affairs in the case of Union Territories.

P a g e 100 | 100

Das könnte Ihnen auch gefallen